Site Loader

Содержание

Закон фарадея направление тока



Закон электромагнитной индукции — закон Фарадея

  • История открытия закона Фарадея
  • Формула закона Фарадея
  • Применение закона Фарадея
  • Примеры решения задач

История открытия закона Фарадея

Майкл Фарадей – английский ученый физик, проводивший опыты с электричеством. До открытия Фарадея считалось, что между магнитным и электрическим полями нет никакой связи. Фарадей обосновал доказательство того, что магнитное поле вызывает электрический ток. Это явление получило название электромагнитной индукции, а закон стал одним из основных в электродинамике.

Фарадей был не единственным ученым, который задумался о связи электрического и магнитного поля. Одновременно с ним над этими явлениями работал Джозеф Генри. Но Фарадей все-таки первым сформулировал и опубликовал результаты своих исследований.

Электромагнитная индукция – появление электрического тока, поля или электрической поляризации в условиях изменения во времени магнитного поля или при движении материальной среды в нем.

К своему открытию Фарадей шёл порядка десяти лет. В 1831 году ученый провел эксперимент, позволивший ему открыть элеткромагнитную индукцию. Он намотал на один железный сердечник две катушки, автономные друг от друга. Одна катушка была подключена к источнику тока, вторая к гальванометру – прибору, измеряющему силу тока. Когда по первой катушке шел электрический ток, ее магнитное поле возрастало. В это же время гальванометр, подключённый ко второй катушке, фиксировал возникновение в ней тока. Таким образом, Фарадей доказал, что в результате влияния магнитного поля, появляется ток. Такой ток стали называть индукционным. Но появлялся он только в момент подключения или отключения первой катушки от цепи. Если по первой катушке шел постоянный ток, то во второй в это время ничего не регистрировалось.

«Электрический магнетизм» проявлялся и в тот момент, когда Фарадей передвигал вторую катушку относительно первой. Сила индукционного тока увеличивалась, если движение катушки было быстрым, и наоборот.

Затем Фарадей заменил первую катушку на магнит, который вводил во вторую катушку. Явление электромагнитной индукции повторилось в точности как в опыте с двумя катушками.

Фарадей пришел к выводу, что возникновение индукционного тока зависит от количества линий магнитного поля, которые проходят сквозь контур.

Формула закона Фарадея

Проводя опыты с катушками и магнитом, Фарадей обнаружил, что величина электродвижущей силы зависит от скорости перемещения катушек или магнита. Это позволило ему выявить закономерность и сформулировать закон электромагнитной индукции.

Закон электромагнитной индукции: электродвижущая сила пропорциональна скорости изменения магнитного потока, проходящего через контур.

Формулировка закона выглядит следующим образом: Ε=-ΔΦ/Δt

E в этой формуле – электродвижущая сила, ΔΦ – изменение магнитного потока, а Δt – время, за которое изменяется магнитный поток. Единицы измерения ЭДС – вольты, магнитного потока – веберы. Δ – разница между конечным и начальным параметром.

В выражении закона Фарадея стоит минус. Он подразумевает применение к этому закону правила Ленца.

Э. Х. Ленц – российский физик, основоположник электротехники. Сформулировал правило индукционного тока в 1833 году.

Правило Ленца добавляет к закону пояснение, что ток, образовавшийся в результате индукции, всегда имеет противоположное направление образующему его магнитному потоку. Магнитное поле индукционного тока всегда препятствует магнитному потоку из внешнего источника. Суть правила очень близка закону сохранения энергии.

Применение закона Фарадея

Закон Фарадея позволил создать такие устройства и приборы как электродвигатели, трансформаторы, генераторы переменного тока, индукционные печи, дроссели, электросчетчики. Принцип действия всех этих устройств основан на электромагнитной индукции.

Трансформаторы передают энергию переменного электромагнитного поля за счет явления взаимной индукции, таким образом трансформируя величину напряжения на разных клеммах.

Примеры решения задач

1. В однородном магнитном поле, индукция которого 1 Тл, имеется плоский проводящий виток, площадь которого равна 100 см 2. Виток расположен перпендикулярно линиям магнитного потока. Сопротивление витка равно 200 мОм. Какой заряд протечет через поперечное сечение витка, если не станет поля?

При исчезновении магнитного поля изменится магнитный поток через виток:

После изменения магнитного потока в контуре появится ЭДС индукции:

2. Концы катушки из тысячи витков радиусом 5 см замкнуты накоротко. Сопротивление катушки 100 Ом. С какой скоростью должна изменяться индукция магнитного поля, перпендикулярного плоскости катушки, чтобы в ней выделялась тепловая мощность 100 мВт.

По закону электромагнитной индукции

3. Провод длиной 2 м складывают пополам и замыкают концы. После этого провод растягивают в квадрат, плоскость которого перпендикулярна силовым линиям магнитного поля с индукцией 64 мкТл. Какое количество электронов пройдет при этом через поперечное сечение провода, если его сопротивление 10 мОм? Вначале площадь контура была равна 0. При растягивании провода в квадрат его площадь стала равна S = a 2 , где a = L/4. Когда изменится площадь, поменяется магнитный поток через контур ΔФ = B ΔS = BL 2 /16.

Источник

Открытие Фарадея и Ленца: закон электромагнитной индукции формула явления

В нашем мире все виды существующих сил, за исключением сил тяготения, представлены электромагнитными взаимодействиями. Во Вселенной, несмотря на удивительное разнообразие воздействий тел друг на друга, в любых веществах, живых организмах всегда встречается проявление электромагнитных сил. Как произошло открытие электромагнитной индукции (ЭИ), расскажем ниже….

  1. Открытие ЭИ
  2. Явление ЭИ
  3. Закон
  4. Другие законы Фарадея
  5. Вихревые токи
  6. Вывод

Открытие ЭИ

Поворот магнитной стрелки вблизи проводника с током в опытах Эрстеда впервые указал на связь электрических и магнитных явлений. Очевидно: электроток «окружает» себя магнитным полем.

Так нельзя ли добиться его возникновения посредством магнитного поля подобную задачу поставил Майкл Фарадей. В 1821 году он отметил это свойство в своем дневнике о превращении магнетизма в электричество.

Успех к ученому пришел не сразу. Лишь глубокая уверенность в единстве природных сил и упорный труд привели его через десять лет к новому великому открытию.

Решение задачи долго не давалось Фарадею и другим его коллегам, потому как они пытались получить электричество в неподвижной катушке, используя действие постоянного магнитного поля. Между тем, впоследствии выяснилось: изменяется количество силовых линий, пронизывающих провода, и возникает электроэнергия.

Явление ЭИ

Процесс появления в катушке электричества в результате изменения магнитного поля характерен для электромагнитной индукции и определяет это понятие. Вполне закономерно, что разновидность тока, возникающего в ходе данного процесса, называется индукционным. Эффект сохранится, если саму катушку оставить без движения, но перемещать при этом магнит. С использованием второй катушки можно и вовсе обойтись без магнита.

Если пропустить электричество через одну из катушек, то при их взаимном перемещении во второй возникнет индукционный ток. Можно надеть одну катушку на другую и менять величину напряжения одной из них, замыкая и размыкая ключ. При этом магнитное поле, пронизывающее катушку, на которую воздействуют ключом, меняется, и это становится причиной возникновения индукционного тока во второй.

Закон

Во время опытов легко обнаружить, что увеличивается число пронизывающих катушку силовых линий стрелка используемого прибора (гальванометр) смещается в одну сторону, уменьшается – в иную. Более тщательное исследование показывает, что сила индукционного тока прямо пропорциональна скорости изменения числа силовых линий. В этом заключен основной закон электромагнитной индукции.

Данный закон выражает формула:

Она применяется, если за период времени t магнитный поток изменяется на одну и ту же величину, когда скорость изменения магнитного потока Ф/t постоянна.

Важно! Для индукционных токов справедлив закон Ома: I=/R, где это ЭДС индукции, которую находят по закону ЭИ.

Замечательные опыты, проведенные когда-то знаменитым английским физиком и ставшие основой открытого им закона, сегодня без особого труда способен проделать любой школьник. Для этих целей используются:

  • магнит,
  • две проволочные катушки,
  • источник электроэнергии,
  • гальванометр.

Закрепим на подставке магнит и поднесем к нему катушку с присоединенными к гальванометру концами.

Поворачивая, наклоняя и перемещая ее вверх и вниз, мы меняем число силовых линий магнитного поля, пронизывающих ее витки.

Гальванометр регистрирует возникновение электричества с постоянно меняющимися в ходе опыта величиной и направлением.

Находящиеся же относительно друг друга в покое катушка и магнит не создадут условий и для возникновения электричества.

Это интересно! Что такое закон всемирного тяготения: формула великого открытия

Другие законы Фарадея

На основе проведенных исследований были сформированы еще два одноименных закона:

  1. Суть первого состоит в такой закономерности: масса вещества m, выделяемая электрическим напряжением на электроде, пропорциональна количеству электричества Q, прошедшему через электролит.
  2. Определение второго закона Фарадея, или зависимости электрохимического эквивалента от атомного веса элемента и его валентности формулируется так: электрохимический эквивалент вещества пропорционален его атомному весу, а также обратно пропорционален валентности.

Из всех существующих видов индукции огромное значение имеет обособленный вид данного явления – самоиндукция. Если мы возьмем катушку, которая имеет большое количество витков, то при замыкании цепи, лампочка загорается не сразу.

На этот процесс может уйти несколько секунд. Очень удивительный на первый взгляд факт. Чтобы понять, в чем здесь дело, необходимо разобраться, что же происходит в момент замыкания цепи. Замкнутая цепь словно «пробуждает» электроток, начинающий свое движение по виткам провода. Одновременно в пространстве вокруг нее мгновенно создается усиливающееся магнитное поле.

Катушечные витки оказываются пронизанными изменяющимся электромагнитным полем, концентрирующимся сердечником. Возбуждаемый же в витках катушки индукционный ток при нарастании магнитного поля (в момент замыкания цепи) противодействует основному. Мгновенное достижение им своего максимального значения в момент замыкания цепи невозможно, оно «растет» постепенно. Вот и объяснение, почему лампочка не вспыхивает сразу. Когда цепь размыкается, основной ток усиливается индукционным в результате явления самоиндукции, и лампочка ярко вспыхивает.

Важно! Суть явления, названного самоиндукцией, характеризуется зависимостью изменения, возбуждающего индукционный ток электромагнитного поля от изменения силы текущего по цепи электротока.

Направление тока самоиндукции определяет правило Ленца. Самоиндукция легко сравнима с инерцией в области механики, поскольку оба явления обладают схожими характеристиками. И действительно, в результате инерции под влиянием силы тело приобретает определенную скорость постепенно, а не сиюминутно. Не сразу – под действием самоиндукции при включении батареи в цепь появляется и электричество. Продолжая сравнение со скоростью, заметим, он так же не способен мгновенно исчезнуть.

Вихревые токи

Наличие вихревых токов в массивных проводниках может послужить еще одним примером электромагнитной индукции.

Специалисты знают, что металлические трансформаторные сердечники, якоря генераторов и электродвигателей никогда не бывают сплошными. При их изготовлении на отдельные тонкие листы, из которых они состоят, накладывается слой лака, изолирующий один лист от другого.

Нетрудно понять, какая сила заставляет человека создавать именно такое устройство. Под действием электромагнитной индукции в переменном магнитном поле сердечник пронизывают силовые линии вихревого электрополя.

Представим, что сердечник изготовлен из сплошного металла. Поскольку его электрическое сопротивление невелико, возникновение индукционного напряжения большой величины было бы вполне объяснимым. Сердечник бы в итоге разогревался, и немалая часть электрической энергии терялась бесполезно. Кроме того, возникла бы необходимость принятия специальных мер для охлаждения. А изолирующие слои не позволяют достигать больших величин.

Индукционные токи, присущие массивным проводникам, называются вихревыми не случайно – их линии замкнуты подобно силовым линиям электрополя, где они и возникают. Чаще всего вихревые токи применяются в работе индукционных металлургических печей для выплавки металлов. Взаимодействуя с породившим их магнитным полем, они иногда становятся причиной занимательных явлений.

Возьмем мощный электромагнит и поместим между вертикально расположенными его полюсами, к примеру, пятикопеечную монету. Вопреки ожиданию, она не упадет, а будет медленно опускаться. Для прохождения нескольких сантиметров ей потребуются секунды.

Поместим, например, пятикопеечную монету между вертикально расположенными полюсами мощного электромагнита и отпустим ее.

Вопреки ожиданию, она не упадет, а будет медленно опускаться. Для прохождения нескольких сантиметров ей потребуются секунды. Передвижение монеты напоминает перемещение тела в вязкой среде. Почему такое происходит.

По правилу Ленца направления возникающих при передвижении монеты вихревых токов в неоднородном магнитном поле таковы, что поле магнита выталкивает монету вверх. Эту особенность используют для «успокоения» стрелки в измерительных приборах. Алюминиевая пластина, находящаяся между магнитными полюсами, прикрепляется к стрелке, и вихревые токи, возникающие в ней, способствуют быстрому затуханию колебаний.

Демонстрацию явления электромагнитной индукции поразительной красоты предложил профессор Московского университета В.К. Аркадьев. Возьмем свинцовую чашу, обладающую сверхпроводящей способностью, и попробуем уронить над ней магнит. Он не упадет, а будет словно «парить» над чашей. Объяснение здесь простое: равное нулю электрическое сопротивление сверхпроводника способствует возникновению в нем электричества большой величины, способных сохраняться продолжительное время и «удерживать» магнит над чашей. По правилу Ленца, направление магнитного поля их таково, что отталкивает магнит и не дает ему упасть.

Изучаем физику закон электро-магнитной индукции

Правильна формулировка закона Фарадея

Вывод

Электромагнитные силы – это силы, которые позволяют людям видеть окружающий мир и чаще других встречаются в природе, например, свет — тоже пример электромагнитных явлений. Жизнь человечества невозможно представить без данного явления.

Это интересно! Специальная теория относительности Эйнштейна: кратко и простыми словами

Источник

Электромагнитная индукция

Явление электромагнитной индукции

Электромагнитная индукция – явление возникновения тока в замкнутом проводящем контуре при изменении магнитного потока, пронизывающего его.

Явление электромагнитной индукции было открыто М. Фарадеем.

  • На одну непроводящую основу были намотаны две катушки: витки первой катушки были расположены между витками второй. Витки одной катушки были замкнуты на гальванометр, а второй – подключены к источнику тока. При замыкании ключа и протекании тока по второй катушке в первой возникал импульс тока. При размыкании ключа также наблюдался импульс тока, но ток через гальванометр тек в противоположном направлении.
  • Первая катушка была подключена к источнику тока, вторая, подключенная к гальванометру, перемещалась относительно нее. При приближении или удалении катушки фиксировался ток.
  • Катушка замкнута на гальванометр, а магнит движется – вдвигается (выдвигается) – относительно катушки.

Опыты показали, что индукционный ток возникает только при изменении линий магнитной индукции. Направление тока будет различно при увеличении числа линий и при их уменьшении.

Сила индукционного тока зависит от скорости изменения магнитного потока. Может изменяться само поле, или контур может перемещаться в неоднородном магнитном поле.

Объяснения возникновения индукционного тока

Ток в цепи может существовать, когда на свободные заряды действуют сторонние силы. Работа этих сил по перемещению единичного положительного заряда вдоль замкнутого контура равна ЭДС. Значит, при изменении числа магнитных линий через поверхность, ограниченную контуром, в нем появляется ЭДС, которую называют ЭДС индукции.

Электроны в неподвижном проводнике могут приводиться в движение только электрическим полем. Это электрическое поле порождается изменяющимся во времени магнитным полем. Его называют вихревым электрическим полем. Представление о вихревом электрическом поле было введено в физику великим английским физиком Дж. Максвеллом в 1861 году.

Свойства вихревого электрического поля:

  • источник – переменное магнитное поле;
  • обнаруживается по действию на заряд;
  • не является потенциальным;
  • линии поля замкнутые.

Работа этого поля при перемещении единичного положительного заряда по замкнутому контуру равна ЭДС индукции в неподвижном проводнике.

Магнитный поток

Магнитным потоком через площадь ​ \( S \) ​ контура называют скалярную физическую величину, равную произведению модуля вектора магнитной индукции ​ \( B \) ​, площади поверхности ​ \( S \) ​, пронизываемой данным потоком, и косинуса угла ​ \( \alpha \) ​ между направлением вектора магнитной индукции и вектора нормали (перпендикуляра к плоскости данной поверхности):

Обозначение – ​ \( \Phi \) ​, единица измерения в СИ – вебер (Вб).

Магнитный поток в 1 вебер создается однородным магнитным полем с индукцией 1 Тл через поверхность площадью 1 м 2 , расположенную перпендикулярно вектору магнитной индукции:

Магнитный поток можно наглядно представить как величину, пропорциональную числу магнитных линий, проходящих через данную площадь.

В зависимости от угла ​ \( \alpha \) ​ магнитный поток может быть положительным ( \( \alpha \) \( \alpha \) > 90°). Если \( \alpha \) = 90°, то магнитный поток равен 0.

Изменить магнитный поток можно меняя площадь контура, модуль индукции поля или расположение контура в магнитном поле (поворачивая его).

В случае неоднородного магнитного поля и неплоского контура магнитный поток находят как сумму магнитных потоков, пронизывающих площадь каждого из участков, на которые можно разбить данную поверхность.

Закон электромагнитной индукции Фарадея

Закон электромагнитной индукции (закон Фарадея):

ЭДС индукции в замкнутом контуре равна и противоположна по знаку скорости изменения магнитного потока через поверхность, ограниченную контуром:

Знак «–» в формуле позволяет учесть направление индукционного тока. Индукционный ток в замкнутом контуре имеет всегда такое направление, чтобы магнитный поток поля, созданного этим током сквозь поверхность, ограниченную контуром, уменьшал бы те изменения поля, которые вызвали появление индукционного тока.

Если контур состоит из ​ \( N \) ​ витков, то ЭДС индукции:

Сила индукционного тока в замкнутом проводящем контуре с сопротивлением ​ \( R \) ​:

При движении проводника длиной ​ \( l \) ​ со скоростью ​ \( v \) ​ в постоянном однородном магнитном поле с индукцией ​ \( \vec \) ​ ЭДС электромагнитной индукции равна:

где ​ \( \alpha \) ​ – угол между векторами ​ \( \vec \) ​ и \( \vec \) .

Возникновение ЭДС индукции в движущемся в магнитном поле проводнике объясняется действием силы Лоренца на свободные заряды в движущихся проводниках. Сила Лоренца играет в этом случае роль сторонней силы.

Движущийся в магнитном поле проводник, по которому протекает индукционный ток, испытывает магнитное торможение. Полная работа силы Лоренца равна нулю.

Количество теплоты в контуре выделяется либо за счет работы внешней силы, которая поддерживает скорость проводника неизменной, либо за счет уменьшения кинетической энергии проводника.

Важно!
Изменение магнитного потока, пронизывающего замкнутый контур, может происходить по двум причинам:

  • магнитный поток изменяется вследствие перемещения контура или его частей в постоянном во времени магнитном поле. Это случай, когда проводники, а вместе с ними и свободные носители заряда, движутся в магнитном поле;
  • вторая причина изменения магнитного потока, пронизывающего контур, – изменение во времени магнитного поля при неподвижном контуре. В этом случае возникновение ЭДС индукции уже нельзя объяснить действием силы Лоренца. Явление электромагнитной индукции в неподвижных проводниках, возникающее при изменении окружающего магнитного поля, также описывается формулой Фарадея.

Таким образом, явления индукции в движущихся и неподвижных проводниках протекают одинаково, но физическая причина возникновения индукционного тока оказывается в этих двух случаях различной:

  • в случае движущихся проводников ЭДС индукции обусловлена силой Лоренца;
  • в случае неподвижных проводников ЭДС индукции является следствием действия на свободные заряды вихревого электрического поля, возникающего при изменении магнитного поля.

Правило Ленца

Направление индукционного тока определяется по правилу Ленца: индукционный ток, возбуждаемый в замкнутом контуре при изменении магнитного потока, всегда направлен так, что создаваемое им магнитное поле препятствует изменению магнитного потока, вызывающего индукционный ток.

Алгоритм решения задач с использованием правила Ленца:

  • определить направление линий магнитной индукции внешнего магнитного поля;
  • выяснить, как изменяется магнитный поток;
  • определить направление линий магнитной индукции магнитного поля индукционного тока: если магнитный поток уменьшается, то они сонаправлены с линиями внешнего магнитного поля; если магнитный поток увеличивается, – противоположно направлению линий магнитной индукции внешнего поля;
  • по правилу буравчика, зная направление линий индукции магнитного поля индукционного тока, определить направление индукционного тока.

Правило Ленца имеет глубокий физический смысл – оно выражает закон сохранения энергии.

Самоиндукция

Самоиндукция – это явление возникновения ЭДС индукции в проводнике в результате изменения тока в нем.

При изменении силы тока в катушке происходит изменение магнитного потока, создаваемого этим током. Изменение магнитного потока, пронизывающего катушку, должно вызывать появление ЭДС индукции в катушке.

В соответствии с правилом Ленца ЭДС самоиндукции препятствует нарастанию силы тока при включении и убыванию силы тока при выключении цепи.

Это приводит к тому, что при замыкании цепи, в которой есть источник тока с постоянной ЭДС, сила тока устанавливается через некоторое время.

При отключении источника ток также не прекращается мгновенно. Возникающая при этом ЭДС самоиндукции может превышать ЭДС источника.

Явление самоиндукции можно наблюдать, собрав электрическую цепь из катушки с большой индуктивностью, резистора, двух одинаковых ламп накаливания и источника тока. Резистор должен иметь такое же электрическое сопротивление, как и провод катушки.

Опыт показывает, что при замыкании цепи электрическая лампа, включенная последовательно с катушкой, загорается несколько позже, чем лампа, включенная последовательно с резистором. Нарастанию тока в цепи катушки при замыкании препятствует ЭДС самоиндукции, возникающая при возрастании магнитного потока в катушке.

При отключении источника тока вспыхивают обе лампы. В этом случае ток в цепи поддерживается ЭДС самоиндукции, возникающей при убывании магнитного потока в катушке.

ЭДС самоиндукции ​ \( \varepsilon_ \) ​, возникающая в катушке с индуктивностью ​ \( L \) ​, по закону электромагнитной индукции равна:

ЭДС самоиндукции прямо пропорциональна индуктивности катушки и скорости изменения силы тока в катушке.

Индуктивность

Электрический ток, проходящий по проводнику, создает вокруг него магнитное поле. Магнитный поток ​ \( \Phi \) ​ через контур из этого проводника пропорционален модулю индукции ​ \( \vec \) ​ магнитного поля внутри контура, а индукция магнитного поля, в свою очередь, пропорциональна силе тока в проводнике.

Следовательно, магнитный поток через контур прямо пропорционален силе тока в контуре:

Индуктивность – коэффициент пропорциональности ​ \( L \) ​ между силой тока ​ \( I \) ​ в контуре и магнитным потоком ​ \( \Phi \) ​, создаваемым этим током:

Индуктивность зависит от размеров и формы проводника, от магнитных свойств среды, в которой находится проводник.

Единица индуктивности в СИ – генри (Гн). Индуктивность контура равна 1 генри, если при силе постоянного тока 1 ампер магнитный поток через контур равен 1 вебер:

Можно дать второе определение единицы индуктивности: элемент электрической цепи обладает индуктивностью в 1 Гн, если при равномерном изменении силы тока в цепи на 1 ампер за 1 с в нем возникает ЭДС самоиндукции 1 вольт.

Энергия магнитного поля

При отключении катушки индуктивности от источника тока лампа накаливания, включенная параллельно катушке, дает кратковременную вспышку. Ток в цепи возникает под действием ЭДС самоиндукции.

Источником энергии, выделяющейся при этом в электрической цепи, является магнитное поле катушки.

Для создания тока в контуре с индуктивностью необходимо совершить работу на преодоление ЭДС самоиндукции. Энергия магнитного поля тока вычисляется по формуле:

Основные формулы раздела «Электромагнитная индукция»

Алгоритм решения задач по теме «Электромагнитная индукция»:

1. Внимательно прочитать условие задачи. Установить причины изменения магнитного потока, пронизывающего контур.

2. Записать формулу:

  • закона электромагнитной индукции;
  • ЭДС индукции в движущемся проводнике, если в задаче рассматривается поступательно движущийся проводник; если в задаче рассматривается электрическая цепь, содержащая источник тока, и возникающая на одном из участков ЭДС индукции, вызванная движением проводника в магнитном поле, то сначала нужно определить величину и направление ЭДС индукции. После этого задача решается по аналогии с задачами на расчет цепи постоянного тока с несколькими источниками.

3. Записать выражение для изменения магнитного потока и подставить в формулу закона электромагнитной индукции.

4. Записать математически все дополнительные условия (чаще всего это формулы закона Ома для полной цепи, силы Ампера или силы Лоренца, формулы кинематики и динамики).

5. Решить полученную систему уравнений относительно искомой величины.

Источник

Закон электромагнитной индукции

О чем эта статья:

Магнитный поток

Прежде, чем разобраться с тем, что такое электромагнитная индукция, нужно определить такую сущность, как магнитный поток.

Представьте, что вы взяли обруч в руки и вышли на улицу в ливень. Чем сильнее ливень, тем больше через этот обруч пройдет воды — поток воды больше.

Если обруч расположен горизонтально, то через него пройдет много воды. А если начать его поворачивать — уже меньше, потому что он расположен не под прямым углом к вертикали.2]

n — вектор нормали (перпендикуляр к поверхности) [-]

Магнитный поток можно наглядно представить как величину, пропорциональную числу магнитных линий, проходящих через данную площадь.

В зависимости от угла ​α магнитный поток может быть положительным (α 90°). Если α = 90°, то магнитный поток равен 0. Это зависит от величины косинуса угла.

Изменить магнитный поток можно меняя площадь контура, модуль индукции поля или расположение контура в магнитном поле (поворачивая его).

В случае неоднородного магнитного поля и неплоского контура, магнитный поток находят как сумму магнитных потоков, пронизывающих площадь каждого из участков, на которые можно разбить данную поверхность.

Ученики Skysmart не боятся сложных понятий по физике и чувствуют себя уверенее на контрольных в школе. А еще — не могут оторваться от домашки: захватывает не хуже, чем тик-ток.

Запишите ребенка на вводное занятие: покажем, как все проходит на интерактивной платформе и вдохновим на учебу!

Электромагнитная индукция

Электромагнитная индукция — явление возникновения тока в замкнутом проводящем контуре при изменении магнитного потока, пронизывающего его.

Явление электромагнитной индукции было открыто М. Фарадеем.

Майкл Фарадей провел ряд опытов, которые помогли открыть явление электромагнитной индукции.

Опыт раз. На одну непроводящую основу намотали две катушки: витки первой катушки были расположены между витками второй. Витки одной катушки были замкнуты на гальванометр, а второй — подключены к источнику тока.

При замыкании ключа и протекании тока по второй катушке в первой возникал импульс тока. При размыкании ключа также наблюдался импульс тока, но ток через гальванометр тек в противоположном направлении.

Опыт два. Первую катушку подключили к источнику тока, а вторую — к гальванометру. При этом вторая катушка перемещалась относительно первой. При приближении или удалении катушки фиксировался ток.

Опыт три. Катушка замкнута на гальванометр, а магнит движется вдвигается (выдвигается) относительно катушки

Вот, что показали эти опыты:

Почему возникает индукционный ток?

Ток в цепи может существовать, когда на свободные заряды действуют сторонние силы. Работа этих сил по перемещению единичного положительного заряда вдоль замкнутого контура равна ЭДС.

Значит, при изменении числа магнитных линий через поверхность, ограниченную контуром, в нем появляется ЭДС, которую называют ЭДС индукции.

Закон электромагнитной индукции

Закон электромагнитной индукции (закон Фарадея) звучит так:

ЭДС индукции в замкнутом контуре равна и противоположна по знаку скорости изменения магнитного потока через поверхность, ограниченную контуром.

Математически его можно описать формулой:

Закон Фарадея

Ɛi — ЭДС индукции [В]

ΔФ/Δt — скорость изменения магнитного потока [Вб/с]

Знак «–» в формуле позволяет учесть направление индукционного тока. Индукционный ток в замкнутом контуре всегда направлен так, чтобы магнитный поток поля, созданного этим током сквозь поверхность, ограниченную контуром, уменьшал бы те изменения поля, которые вызвали появление индукционного тока.

Если контур состоит из ​N витков (то есть он — катушка), то ЭДС индукции будет вычисляться следующим образом.

Закон Фарадея для контура из N витков

Ɛi — ЭДС индукции [В]

ΔФ/Δt — скорость изменения магнитного потока [Вб/с]

N — количество витков [-]

Сила индукционного тока в замкнутом проводящем контуре с сопротивлением ​R​:

Закон Ома для проводящего контура

Ɛi — ЭДС индукции [В]

I — сила индукционного тока [А]

R — сопротивление контура [Ом]

Если проводник длиной l будет двигаться со скоростью ​v​ в постоянном однородном магнитном поле с индукцией ​B​ ЭДС электромагнитной индукции равна:

ЭДС индукции для движущегося проводника

Ɛi — ЭДС индукции [В]

B — магнитная индукция [Тл]

v — скорость проводника [м/с]

l — длина проводника [м]

Возникновение ЭДС индукции в движущемся в магнитном поле проводнике объясняется действием силы Лоренца на свободные заряды в движущихся проводниках. Сила Лоренца играет в этом случае роль сторонней силы.

Движущийся в магнитном поле проводник, по которому протекает индукционный ток, испытывает магнитное торможение. Полная работа силы Лоренца равна нулю.

Количество теплоты в контуре выделяется либо за счет работы внешней силы, которая поддерживает скорость проводника неизменной, либо за счет уменьшения кинетической энергии проводника.

Изменение магнитного потока, пронизывающего замкнутый контур, может происходить по двум причинам:

  • вследствие перемещения контура или его частей в постоянном во времени магнитном поле. Это случай, когда проводники, а вместе с ними и свободные носители заряда, движутся в магнитном поле
  • вследствие изменения во времени магнитного поля при неподвижном контуре. В этом случае возникновение ЭДС индукции уже нельзя объяснить действием силы Лоренца. Явление электромагнитной индукции в неподвижных проводниках, возникающее при изменении окружающего магнитного поля, также описывается формулой Фарадея

Таким образом, явления индукции в движущихся и неподвижных проводниках протекают одинаково, но физическая причина возникновения индукционного тока оказывается в этих двух случаях различной:

  • в случае движущихся проводников ЭДС индукции обусловлена силой Лоренца
  • в случае неподвижных проводников ЭДС индукции является следствием действия на свободные заряды вихревого электрического поля, возникающего при изменении магнитного поля.

Правило Ленца

Чтобы определить направление индукционного тока, нужно воспользоваться правилом Ленца.

Академически это правило звучит следующим образом: индукционный ток, возбуждаемый в замкнутом контуре при изменении магнитного потока, всегда направлен так, что создаваемое им магнитное поле препятствует изменению магнитного потока, вызывающего индукционный ток.

Давайте попробуем чуть проще: катушка в данном случае — это недовольная бабуля. Забирают у нее магнитный поток — она недовольна и создает магнитное поле, которое этот магнитный поток хочет обратно отобрать.

Дают ей магнитный поток, забирай, мол, пользуйся, а она такая — «Да зачем сдался мне ваш магнитный поток!» и создает магнитное поле, которое этот магнитный поток выгоняет.

Источник

Электромагнитная индукция. Опыты Фарадея. Закон электромагнитной индукции

ИСХОДНЫЙ МАТЕРИАЛ

План изложения учебного материала по теме

«Электромагнитная индукция. Опыты Фарадея. Закон электромагнитной индукции»

1.     История открытия явления.

2.     Опыты Фарадея.

3.     Изменение направления индукционного тока.

4.     Закон электромагнитной индукции. Магнитный поток.

5.     Возникновение вихревого магнитного поля.

1.     История открытия явления.

До середины XIX века считалось, что электрическое и магнитное поле не имеют никакой связи, и природа их существования различна. Но М. Фарадей был уверен в единой природе этих полей и их свойств. Явление электромагнитной индукции, обнаруженное им, впоследствии стало фундаментом для устройства генераторов всех электростанций. Благодаря этому открытию знания человечества о электромагнетизме шагнули далеко вперед.

Фарадей проделал следующий опыт: он замыкал цепь в катушке I и вокруг нее возрастало магнитное поле. Далее линии индукции данного магнитного поля пересекали катушку II, в которой возникал индукционный ток.

Рис. 1. Схема опыта Фарадея


На самом деле, одновременно с Фарадеем, но независимо от него, другой ученый Джозеф Генриобнаружил это явление. Однако Фарадей опубликовал свои исследования раньше. Таким образом, автором закона электромагнитной индукции стал Майкл Фарадей.

Сколько бы экспериментов не проводил Фарадей, неизменным оставалось одно условие: для образования индукционного тока важным является изменение магнитного потока, пронизывающего замкнутый проводящий контур (катушку).

1.     Опыты Фарадея.

Для проведения опыта он взял катушку с большим количеством витков и присоединил ее к миллиамперметру (прибору, измеряющему силу тока). По направлению вверх и вниз ученый передвигал магнит по катушке.

Во время проведения эксперимента, в катушке действительно появлялся электрический ток по причине изменения магнитного поля вокруг нее.

По наблюдениям Фарадея стрелка миллиамперметра отклонялась и указывала на то, что движение магнита порождает собой электрический ток. При остановке магнита стрелка показывала нулевую разметку, т.е. ток не циркулировал по цепи.

Рис. 2 Изменение силы тока в катушке за счет передвижения реостата.

Данное явление, при котором  ток возникает под действием переменного магнитного поля в проводнике, назвали явлением электромагнитной индукции.

1.     Изменение направления индукционного тока.

В своих последующих исследованиях Майкл Фарадей пытался выяснить, что влияет на направление возникающего индукционного электрического тока. Проводя опыты, он заметил, что изменяя числа мотков на катушке или полярность магнитов, направление электрического тока, которое возникает в замкнутой сети, меняется.

1.     Закон электромагнитной индукции. Магнитный поток.

Явление электромагнитной индукции определяется возникновением электрического тока в замкнутом электропроводящем контуре при изменении магнитного потока через площадь этого контура.

Основной закон Фарадея заключается в том, что электродвижущая сила (ЭДС) прямо пропорциональна скорости изменения магнитного потока. Определение магнитного потока:

Магнитным потоком Ф через поверхность S называют количество линий вектора магнитной индукции B, проходящих через поверхность S.

Формула магнитного потока:

Ф = BS cosα , здесь α — угол между направлением вектора магнитной индукции B и нормалью к поверхности S.

Из формулы магнитного потока видно, что максимальным магнитный поток будет при cos α = 1, а это случится, когда вектор B параллелен нормали к поверхности S. Минимальным магнитный поток будет при cos α = 0, это будет, когда вектор B перпендикулярен нормали к поверхности S, ведь в этом случае линии вектора B будут скользить по поверхности S, не пересекая её.

А по определению магнитного потока учитываются только те линии вектора магнитной индукции, которые пересекают данную поверхность.

Измеряется магнитный поток в веберах (вольт-секундах): 1 вб = 1 в * с. Кроме того, для измерения магнитного потока применяют максвелл: 1 вб = 108 мкс. Соответственно 1 мкс = 10-8 вб.

Магнитный поток является скалярной величиной.

Формула закона электромагнитной индукции Фарадея выглядит следующим образом:

Рис. 3. Формула закона электромагнитной индукции

И если сама формула, исходя из вышесказанных объяснений не порождает вопросов, то знак «-» может вызвать сомнения. Оказывается, существует правило Ленца – русского ученого, который проводил свои исследования, основываясь на постулатах Фарадея. По Ленцу знак «-» указывает на направление возникающей ЭДС, т.е. индукционный ток направлен так, что магнитный поток, который он создает, через площадь, ограниченную контуром, стремится препятствовать тому изменению потока, которое вызывает данный ток.

Закон Фарадея-Максвелла

В 1873 году Дж. К. Максвелл по-новому изложил теорию электромагнитного поля. Уравнения, которые он вывел, легли в основу современной радиотехники и электротехники. Они выражаются следующим образом:

Edl = -dФ/dt – уравнение электродвижущей силы

Hdl = -dN/dt – уравнение магнитодвижущей силы.

Где E – напряженность электрического поля на участке dl; H – напряженность магнитного поля на участке dl; N – поток электрической индукции, t – время.

1.     Возникновение вихревого магнитного поля.

1 Вб = 1 Тл · 1 м2.

Симметричный характер данных уравнений устанавливает связь электрических и магнитных явлений, а также магнитных с электрическими. Физический смысл, которым определяются эти уравнения, можно выразить следующими положениями:

  • если электрическое поле изменяется, то это изменение всегда сопровождается магнитным полем.
  • если магнитное поле изменяется, то это изменение всегда сопровождается электрическим полем.

Рис. 4. Возникновение вихревого магнитного поля


Также Максвелл установил, что распространение электромагнитного поля равна скорости распространения света.

ПРИЛОЖЕНИЕ №1

КОНТРОЛЬ ЗНАНИЙ ПО ПРЕДЫДУЩЕЙ ТЕМЕ (устно)

«Сила Лоренца. Движение заряженной частицы в магнитных и электрических полях»

1.     Сформулируйте понятие и выведите формулу для нахождения силы Лоренца.

Ответ: Сила Ампера, действующая на отрезок проводника длиной Δl с силой тока I, находящийся в магнитном поле B, может быть выражена через силы, действующие на отдельные носители заряда.

Пусть концентрация носителей свободного заряда в проводнике есть n, а q – заряд носителя. Тогда произведение n q υ S, где υ – модуль скорости упорядоченного движения носителей по проводнику, а S – площадь поперечного сечения проводника, равно току, текущему по проводнику: 

Выражение для силы Ампера можно записать в виде: 

F = q n 

S Δl υB sin α.

Так как полное число N носителей свободного заряда в проводнике длиной Δl и сечением S равно n S Δl, то сила, действующая на одну заряженную частицу, равна 

FЛ = q υ B sin α.

Эту силу называют силой Лоренца. Угол α в этом выражении равен углу между скоростью  и вектором магнитной индукции  .

2.     Сформулируйте правило левой руки для нахождения силы Лоренца.

Ответ: Направление силы Лоренца, действующей на положительно заряженную частицу, так же, как и направление силы Ампера, может быть найдено по правилу левой руки или по правилу буравчика.


 ПРАВИЛО ЛЕВОЙ РУКИ для заряженной частицы с целью определения направления силы, действующей на отдельную заряженную частицу, движущуюся в магнитном поле. Если ЛЕВУЮ РУКУ расположить так, чтобы линии магнитного поля входили в ладонь перпендикулярно к ней, а четыре пальца были направлены по движению положительно заряженной частицы (или против движения отрицательно заряженной частицы), то отставленный на 90 градусов большой палец покажет направление действующей на частицу силы.

Взаимное расположение векторов ,  и  для положительно заряженной частицы показано на рисунке.

 Модуль силы Лоренца  численно равен площади параллелограмма, построенного на векторах  и  помноженной на заряд q

Сила Лоренца направлена перпендикулярно векторам   и .

3.     Опишите движение заряженной частицы в магнитном поле.

Ответ: При движении заряженной частицы в магнитном поле сила Лоренца работы не совершает. Поэтому модуль вектора скорости при движении частицы не изменяется.

Если заряженная частица движется в однородном магнитном поле под действием силы Лоренца, а ее скорость  лежит в плоскости, перпендикулярной вектору  то частица будет двигаться по окружности радиуса 

Сила Лоренца в этом случае играет роль центростремительной силы.

Круговое движение заряженной частицы в однородном магнитном поле

Период обращения частицы в однородном магнитном поле равен 

Это выражение показывает, что для заряженных частиц заданной массы m период обращения не зависит от скорости υ и радиуса траектории R.

Угловая скорость движения заряженной частицы по круговой траектории 

называется циклотронной частотой. Циклотронная частота не зависит от скорости (следовательно, и от кинетической энергии) частицы. Это обстоятельство используется в циклотронах – ускорителях тяжелых частиц (протонов, ионов). Принципиальная схема циклотрона приведена на рисунке.

Движение заряженных частиц в вакуумной камере циклотрона.

4.     Опишите движение заряженной частицы в электрическом поле.

Ответ: Электрическое поле создается между пластинами плоского конденсатора, магнитное поле – в зазоре между полюсами электромагнита. Начальная скорость  заряженных частиц направлена перпендикулярно векторам  и 

На частицу, движущуюся в скрещенных электрическом и магнитном полях, действуют электрическая сила  и  магнитная сила Лоренца. При условии E = υB. эти силы точно уравновешивают друг друга. Если это условие выполняется, частица будет двигаться равномерно и прямолинейно и, пролетев через конденсатор, пройдет через отверстие в экране. При заданных значениях электрического и магнитного полей селектор выделит частицы, движущиеся со скоростью .

Далее частицы с одним и тем же значением скорости попадают в камеру масс-спектрометра, в которой создано однородное магнитное поле  Частицы движутся в камере в плоскости, перпендикулярной магнитному полю, под действием силы Лоренца. Траектории частиц представляют собой окружности радиусов   Измеряя радиусы траекторий при известных значениях υ и B’ можно определить отношение . В случае изотопов (q1 = q2) масс-спектрометр позволяет разделить частицы с разными массами.

Современные масс-спектрометры позволяют измерять массы заряженных частиц с точностью выше 10–4.

Критерии оценки:

Оценка «5» — на поставленный вопрос студент дал полный развернутый ответ и ответил на дополнительный вопрос;

Оценка «4» — на поставленный вопрос студент дал полный развернутый ответ, но не ответил на дополнительный вопрос;

Оценка «3» — на поставленный вопрос студент дал неполный ответ и не смог ответить на дополнительный вопрос;

Оценка «2» — не ответил на поставленный вопрос.

ПРИЛОЖЕНИЕ №2

ЗАДАНИЯ ДЛЯ ЗАКРЕПЛЕНИЯ И СИСТЕМАТИЗАЦИИ НОВЫХ ЗНАНИЙ (письменно, не оценивается)

Физика 11 Разноуровневые самостоятельные и контрольные работы А. Кирик стр.4 начальный уровень №1-6, стр. 6 достаточный уровень №1, 2, 3.

Эталоны ответов к заданиям для закрепления и систематизации

Уровень /№

1

2

3

4

5

6

Начальный уровень

Б, В

В

Б, В

Б

А, Б

В

Достаточный уровень

1

2

3

6 м/с

0,5 м

0,3 Тл

ПРИЛОЖЕНИЕ № 3

ЗАДАНИЯ ДЛЯ ПРЕДВАРИТЕЛЬНОГО КОНТРОЛЯ ЗНАНИЙ

(Устно, не оценивается. Эталоны ответов к вопросам для предварительного контроля знаний содержатся в исходном материале)

1.     Расскажите об истории открытия явления.

2.     Какие еще ученые занимались данным явлением?

3.     Сформулируйте определение электромагнитной индукции.

4.     Опишите опыты Фарадея.

5.     От каких факторов зависит направление индукционного тока?

6.     От чего зависит сила возникающего индукционного тока?

7.     Дайте определение магнитного потока.

8.     При каком условии магнитный поток, пронизывающий контур, будет максимальным?

9.     Сформулируйте закон электромагнитной индукции.

10. На что указывает знак «-» в формуле закона электромагнитной индукции?

11. По какому правилу можно определить направление индукционного тока?

12. Почему возникающее магнитное поле называется вихревым?

ПРИЛОЖЕНИЕ №4

КОНТРОЛИРУЮЩИЙ МАТЕРИАЛ (письменно)

Тест

1.     Кто одновременно с Фарадеем открыл явление электромагнитной индукции?

1.      Джозеф Генри

2.      Э.Х. Ленц

3.      Дж.К. Максвелл

4.      Ш. Кулон

 2.В каком веке было открыто явление электромагнитной индукции?

1.      в XVII

2.      в XVIII

3.      в XIX

4.      в XX

3. Фарадей был уверен, что электрическое и магнитное поле:

1.      не имеют ни какой связи друг с другом

2.      имеют общую природу

3.      ничем не отличаются друг от друга

4.      ни один из ответов не является правдой

4.Для образования индукционного тока необходимо:

1.      изменение магнитного потока

2.      хаотичное движение молекул

3.      неизменность магнитного потока

4.      ни одно из утверждений ни является верным

1.     По закону Фарадея ЭДС:

1.      равна скорости изменения магнитного потока

2.      прямо пропорциональна скорости изменения магнитного потока

3.      обратно пропорциональна скорости изменения магнитного потока

4.      не зависит от скорости изменения магнитного потока

6. Знак «-» в формуле закона электромагнитной индукции указывает:

1.      на направление тока

2.      на уменьшение времени изменения магнитного потока

3.      на направление возникающей ЭДС

4.      ни одно из утверждений не является верным

7. Явление электромагнитной индукции стало:

1.      фундаментом для устройства генераторов всех электростанций

2.      основой для устройства ветряных мельниц

3.      основой для устройства гидростанций

4.      ни одно из утверждений не является верным

8. Скорость распространения электромагнитного поля:

1.      больше скорости света

2.      меньше скорости света

3.      равна скорости распространения света

4.      ни одно из утверждений не является верным

9. При электромагнитной индукции ток возникает:

1.      в замкнутом электропроводящем контуре

2.      в незамкнутом контуре

3.      вне контура

4.      ни одно из утверждений не является верным

10. В каком году Максвелл по-новому изложил теорию электромагнитного поля?

1.      в 1879

2.      в 1765

3.      в 1873

4.      в 1901


Эталоны ответов к заданиям контролирующего материала:

Номер задания

1

2

3

4

5

6

7

8

9

10

Ответы

А

С

В

А

В

С

А

С

А

С

Критерии оценки:

за 7 правильных ответов– «3» балла;

за 8 правильных ответов – «4» балла;

за 9, 10 правильных ответов – «5» баллов.

ЗАДАНИЕ ДЛЯ САМОСТОЯТЕЛЬНОЙ ВНЕАУДИТОРНОЙ РАБОТЫ СТУДЕНТОВ

Цель: Определить объем информации для самостоятельной работы студента, обратить внимание на значимые моменты.

Время для выполнения задания: 45 минут.

Г. Я. Мякишев, Б. Б. Буховцев, Н. Н. Соцкий, Физика. 11 класс. Учебник для общеобразовательных учреждений (с приложением на электронном носителе). Базовый и профильный уровни — М.: Просвещение, 2016 г., с. 31-35, параграфы 7, 8 прочитать, конспект выучить; подготовить сообщение по теме «Майкл Фарадей».

ТРЕБОВАНИЯ К ОФОРМЛЕНИЮ СООБЩЕНИЯ

1. Сообщение оформляется на компьютере, сдается преподавателю в мультифоре.

2. Шрифт TimesNewRoman, 14 пт, межстрочный интервал – одинарный, поля по 1,5 см справа и слева, текст выравниваются по ширине, заголовок – посредине. Ф.И. автора – по правому краю.

3. Объем сообщения – 2-3 страницы формата А4; время выступления – не более 5 минут.

4. В сообщении не выделяются главы; недопустимы орфографические ошибки, опечатки, записи и исправления ручкой или карандашом.

5. В конце сообщения указывается список информационных источников.

6. Сообщение может сопровождаться мультимедийной презентацией (по желанию автора).

!!! Определите самостоятельно, соответствует ли Ваше сообщение требованиям к оформлению. Для этого внимательно прочтите их и подчеркните каждое выполненное требование. Проведите коррекцию работы по тем требованиям, которые не выполнены.

Критерии оценки:

  • студент выучил конспект – «3» балла;
  • студент прочитал параграфы и выучил конспект, владеет информацией из учебника – «4» балла;
  • студент выучил конспект, владеет информацией из учебника, подготовил сообщение, соответствующее требованиям – «5» баллов.

 

 

 

Список использованных источников

1.Батышев С.Я. Профессиональная педагогика: Учебник для студентов, обучающихся по педагогическим специальностям и направлениям /С.Я.Батышев. – М.: Ассоциация «Профессиональное образование»/, 1997. – 512 с.

2.Батышев С.Я. Реформа профессиональной школы: Опыт, поиск, задачи, пути реализации. – М.: Высшая школа, 1987. –340 с.

3. Болдырев Н. И., Гончаров Н. К., Есипов Б. П. и др. Педагогика. — М.: Наука, 1998.

4. Большой энциклопедический словарь. 2-е изд. перераб. и доп. – М.: «Большая Российская энциклопедия»; СПб.: «Норинт», 2007. – 1456 с.

5. Бордовская Н.В., Реан А.А. Педагогика. Учебник для вузов- CПб.: Питер, 2000.

6. Буланова-Топоркова М.В. Педагогика и психология высшей школы: учебное пособие. – Ростов-на-Дону: Феникс, 2002. — 544 с.

7. Подласый И.П. Педагогика: 100 вопросов — 100 ответов: учеб. пособие для вузов/ И. П. Подласый. — М.: ВЛАДОС-пресс, 2004. — 365 с.

8. Общая и профессиональная педагогика: Учебное пособие для студентов, обучающихся по специальности «Профессиональное обучение»: В 2-х книгах /Под ред. В.Д. Симоненко, М.В. Ретивых. — Брянск: Изд-во Брянского государственного университета/, 2003. — Кн.1 — 174 с.

9. Сластенин В.А. и др. Педагогика: Учеб. пособие для студ. высш. пед. учеб. заведений / В. А. Сластенин, И. Ф. Исаев, Е. Н. Шиянов; Под ред. В.А. Сластенина. — М.: Издательский центр «Академия», 2002. — 576 с.

10. Смирнов И.П. Проблемы развития профессионального образования в РФ // Стратегия профессионального образования молодежи. — М., 1995.– С.12-22.

11. Стандарты профессионального образования зарубежных стран. – М.: НПО, 2007.-42с.

12. Харламов И.Ф. Педагогика: Учебное пособие. 2-е изд. — М.: Высшая школа, 1990. -576 с.

13. Закон РФ «О высшем и послевузовском профессиональном образовании» //Бюллетень Госкомитета РФ по высшему образованию. — 1996. — №10. — С. 1-59.

14. Зеер Э.Ф. Психология профессионального образования: Учеб. пособие. -Екатеринбург: Изд-во Урал. гос. проф.-пед. ун-та, 2000. — С. 181-224.

15. Зеер Э.Ф. Психология личностно ориентированного профессионального образования. — Екатеринбург: Изд-во Урал. гос. проф.-пед. ун-та, 2000. — С 3-71.

16. Концепция модернизации российского образования на период до 2010 года. Одобрена Распоряжением Правительства РФ №1756-р от 29 декабря 2001 г. // Официальные документы в образовании. — 2002. — №4. — С. 3-31.

17. Маркова А.К. Психология профессионализма. — М., 1996. — С. 49-57; 262-270.

18. Шелтен А. Введение в профессиональную педагогику: Учеб. пособие. -Екатеринбург: Изд-во Урал. гос. проф-пед. ун-та, 1996. — С. 91-106; 154-164.

19. Новиков A.M. Профессиональное образование в России. — М.: Просвещение, 1997. — 254 с.

20. Семушина Л.Г., Ярошенко Н.Г. Содержание и технологии обучения в средних специальных учебных заведениях: учеб. пособие для преп. учреждений сред. проф. образования — М.: Мастерство, 2001. — 272 с.

21. Беляева А.П.Дидактические принципы профессиональной подготовки в профтехучилищах: Методическое пособие.- М.:Высш.шк.,1991.,с.143-184

22. Коменский Я.А. Избранные педагогические сочинения.-М.:1982г.,с.242-404.

23. Педагогика. Учебное пособие для студентов педагогических вузов и педагогических колледжей / Под ред. И.П. Пидкасистый. — М.: Педагогическое общество России, 1998. — 640с., с.129-192.

24.Кукушин В.С. Введение в педагогическую деятельность: учебное пособие. Изд-е 2-е М.: ИКЦ «МарТ», Рост он/Д, 2005 год

25. Под общ. ред. Н. Л. Селивановой. Гуманистические воспитательные системы вчера и сегодня//Под общ. ред. Н. Л. Селивановой. — М.: Пед. об-во России, 1998 г.—336 с.

26. Симоненко В.Д. – Общая и профессиональная педагогика – М.: Вентана-граф, 2006 год

27. Кларин М. В. Технология обучения: идеал и реальность// Кларин М. В.— Рига, «Эксперимент», 1999 г.—180 с.

Internet

1.www.dist-edu.ru

2.www.hse.ru

3.http://ito.bitpro.ru

4.www.ui.usm.ru

5.http://biro.ufanet.ru

6.http://kampi.kcr.ru2.9 Технология дистанционного обучения

7. http://refleader.ru/jgemerrnaujgmer.html


 

Скачано с www.znanio.ru

Закон электромагнитной индукции Фарадея

Если взять замкнутую проводящую систему и создать в ней условия для того чтобы магнитный поток изменился в магнитном поле, то в результате этих движений появится электрический ток. Данное обстоятельство описывает закон электромагнитной индукции Фарадея – английского ученого, который при проведении опытов добился превращения магнитной энергии в электричество. Оно получило название индукционного, поскольку до того времени его можно было создать лишь гальваническим путем.

История открытия

Явление электромагнитной индукции было открыто сразу двумя учеными. Это были Майкл Фарадей и Джозеф Генри, сделавшие свое открытие в 1831 году. Публикация Фарадеем результатов проведенных экспериментов была сделана раньше его коллеги, поэтому индукцию связывают именно с этим ученым. В дальнейшем это понятие было включено в систему СГС.

Для демонстрации явления использовался железный тор, напоминающий конфигурацию современного трансформатора. Противоположные стороны его были обмотаны двумя проводниками с целью использования электромагнитных свойств.

К одному из проводов подключался ток, вызывающий своеобразную электрическую волну при прохождении сквозь тор, и некоторый электрический всплеск с противоположной стороны. Наличие тока было зафиксировано гальванометром. Точно такой же всплеск электричества наблюдался и в момент отключения провода.

Постепенно были обнаружены и другие формы проявления электромагнитной индукции. Кратковременное возникновение тока наблюдалось во время генерации его на медном диске, вращающемся возле магнита. На самом диске был установлен скользящий электропровод.

Наибольшие представление о том, что такое индуктивность, дал эксперимент с двумя катушками. Одна из них, с меньшими размерами, подключена к жидкостной батарее, расположенной на рисунке с правой стороны. Таким образом, через эту катушку начинает протекать электрический ток, под действием которого возникает магнитное поле.

Когда обе катушки находятся в неподвижном положении относительно друг друга, никаких явлений не происходит. Когда небольшая катушка начинает двигаться, то есть выходить из большой катушки или входить в нее, наступает изменение магнитного потока. В результате, в большой катушке наблюдается появление электродвижущей силы.

Открытие Фарадея доработал другой ученый – Максвелл, который обосновал его математически, отображая данное физическое явление дифференциальными уравнениями. Еще одному ученому-физику – Ленцу удалось определить направление электротока и ЭДС, полученных под действием электромагнитной индукции.

Законы электромагнитной индукции

Сущность электромагнитной индукции определяется замкнутым контуром с электропроводностью, площадь которого пропускает через себя изменяющийся магнитный поток. В этот момент под влиянием магнитного потока появляется электродвижущая сила Еi и в контуре начинает течь электрический ток.

Закон Фарадея для электромагнитной индукции заключается в прямой зависимости ЭДС и скорости, составляющих пропорцию. Данная скорость представляет собой время, в течение которого магнитный поток подвергается изменениям.

Данный закон выражается формулой Еi = – ∆Ф/∆t, в которой Еi – значение электродвижущей силы, возникающей в контуре, а ∆Ф/∆t является скоростью изменения магнитного потока. В этой формуле не совсем понятным остается знак «минус», но ему тоже имеется свое объяснение. В соответствии с правилом русского ученого Ленца, изучавшего открытия Фарадея, этот знак отображает направление ЭДС, возникающей в контуре. То есть, направление индукционного тока происходит таким образом, что создаваемый им магнитный поток на площади, ограниченной контуром, препятствует изменениям, вызванным этим током.

Открытия Фарадея были доработаны Максвеллом, у которого теория электромагнитного поля получила новые направления. В результате, появился закон Фарадея и Максвелла, выраженный в следующих формулах:

  • Edl = -∆Ф/∆t – отображает электродвижущую силу.
  • Hdl = -∆N/∆t – отображает магнитодвижущую силу.

В этих формулах Е соответствует напряженности электрического поля на определенном участке dl, Н является напряженностью магнитного поля на этом же участке, N – поток электрической индукции, t – период времени.

Оба уравнения отличаются симметричностью, позволяющей сделать вывод, что магнитные и электрические явления связаны между собой. С физической точки зрения эти формулы определяют следующее:

  • Изменениям в электрическом поле всегда сопутствует образование магнитного поля.
  • Изменения в магнитном поле всегда происходят одновременно с образованием электрического поля.

Изменяющийся магнитный поток, проходящий сквозь замкнутую конфигурацию проводящего контура, приводит к возникновению в этом контуре электрического тока. Это основная формулировка закона Фарадея. Если изготовить проволочную рамку и поместить ее внутри вращающегося магнита, то в самой рамке появится электричество.

Это и будет индукционный ток, в полном соответствии с теорией и законом Майкла Фарадея. Изменения магнитного потока, проходящего через контур, могут быть произвольными. Следовательно, формула ∆Ф/∆t бывает не только линейной, а в определенных условиях принимает любую конфигурацию. Если изменения происходят линейно, то ЭДС электромагнитной индукции, возникающей в контуре, будет постоянной. Временной интервал t становится каким угодно, а отношение ∆Ф/∆t не будет зависеть от его продолжительности.

Если же изменения магнитного потока принимают более сложную форму, то ЭДС индукции уже не будет постоянной, а будет зависеть от данного промежутка времени. В этом случае временной интервал рассматривается в качестве бесконечно малой величины и тогда соотношение ∆Ф/∆t с точки зрения математики станет производной от изменяющегося магнитного потока.

Существует еще один вариант, трактующий закон электромагнитной индукции Фарадея. Его краткая формулировка объясняет, что действие переменного магнитного поля вызывает появление вихревого электрического поля. Этот же закон можно трактовать как одну из характеристик электромагнитного поля: вектор напряженности поля может циркулировать по любому из контуров со скоростью, равной скорости изменения магнитного потока, проходящего через тот или иной контур.

Закон электромагнитной индукции фарадея для начинающих. SA

В 1821 году Майкл Фарадей записал в своем дневнике: «Превратить магнетизм в электричество». Через 10 лет эта задача была им решена. В 1831 г. Майкл Фарадей установил, что во всяком замкнутом проводящем контуре при изменении потока магнитной индукции через поверхность, ограниченную этим контуром, возникает электрический ток. Это явление называется электромагнитной индукцией , а возникающий ток – индукционным (рис. 3.27).

Рис. 3.27 Опыты Фарадея

Индукционный ток возникает всегда, когда происходит изменение сцепленного с контуром потока магнитной индукции. Сила индукционного тока не зависит от способа изменения потока магнитной индукции, а определяется лишь скоростью его изменения.

Закон Фарадея: сила индукционного тока, возникающего в замкнутом проводящем контуре (ЭДС индукции, возникающая в проводнике), пропорциональна скорости изменения магнитного потока, сцепленного с контуром (проникающего через поверхность, ограниченную контуром), и не зависит от способа изменения магнитного потока.

Ленц установил правило, с помощью которого можно найти направление индукционного тока. Правило Ленца: индукционный ток направлен таким образом, что собственным магнитным полем препятствует изменению внешнего магнитного потока, пересекающего поверхность контура (рис. 3.28).

Рис. 3.28 Иллюстрация правила Ленца

Согласно закону Ома электрический ток в замкнутой цепи может возникать только в том случае, если в этой цепи появится ЭДС. Поэтому обнаруженный Фарадеем индукционный ток свидетельствует о том, что в замкнутом контуре, находящемся в переменном магнитном поле возникает ЭДС индукции. Дальнейшее исследование показало, что ЭДС электромагнитной индукции в контуре пропорционально изменению магнитного потока сквозь поверхность, ограниченную этим контуром.

Мгновенное значение ЭДС индукции выражается законом Фарадея-Ленца )

где – потокосцепление замкнутого проводящего контура.

Открытие явления электромагнитной индукции:

1. показало взаимосвязь между электрическим и магнитным полем;

2. предложило способ получения электрического тока с помощью магнитного поля.

Таким образом, возникновение ЭДС индукции возможно и в случае неподвижного контура , находящегося в переменном магнитном поле. Однако сила Лоренца на неподвижные заряды не действует, поэтому с ее помощью нельзя объяснить возникновение ЭДС индукции.

Опыт показывает, что ЭДС индукции не зависит от рода вещества проводника, от состояния проводника, в частности от его температуры, которая может быть даже неодинаковой вдоль проводника. Следовательно, сторонние силы связаны не с изменением свойств проводника в магнитном поле, а обусловлены самим магнитным полем.

Английский физик Максвелл для объяснения ЭДС индукции в неподвижных проводниках предположил, что переменное магнитное поле возбуждает в окружающем пространстве вихревое электрическое поле , которое и является причиной возникновения индукционного тока в проводнике. Вихревое электрическое поле не является электростатическим (т. е. потенциальным).

ЭДС электромагнитной индукции возникает не только в замкнутом проводнике с током, но и в отрезке проводника, пересекающем при своем движении линии магнитной индукции (рис. 3.29).

Рис. 3.29 Образование ЭДС индукции в движущемся проводнике

Пусть прямолинейный отрезок проводника длиной l движется слева направо скоростью v (рис. 3.29). Индукция магнитного поля В направлена от нас. Тогда на электроны, движущиеся со скоростью v действует сила Лоренца

Под действием этой силы электроны будут смещаться к одному из концов проводника. Следовательно, возникает разность потенциалов и электрическое поле внутри проводника с напряженностью E . Со стороны возникшего электрического поля на электроны будет действовать сила qE , направление которой противоположно силе Лоренца. Когда эти силы уравновесят друг друга, то движение электронов прекратится.

Цепь разомкнута, значит , но в проводнике нет гальванического элемента или других источников тока, значит, это будет ЭДС индукции

.

При перемещении в магнитном поле замкнутого проводящего контура ЭДС индукции находится во всех его участках, пересекающих линии магнитной индукции. Алгебраическая сумма этих ЭДС равна общей ЭДС индукции замкнутого контура.

Для описания процессов в физике и химии есть целый ряд законов и соотношений, полученных экспериментальным и расчетным путем. Ни единого исследования нельзя провести без предварительной оценки процессов по теоретическим соотношениям. Законы Фарадея применяются и в физике, и в химии, а в этой статье мы постараемся кратко и понятно рассказать о всех знаменитых открытиях этого великого ученого.

История открытия

Закон Фарадея в электродинамике был открыт двумя ученными: Майклом Фарадеем и Джозефом Генри, но Фарадей опубликовал результаты своих работ раньше – в 1831 году.

В своих демонстрационных экспериментах в августе 1831 г. он использовал железный тор, на противоположные концы которого был намотан провод (по одному проводу на стороны). На концы одного первого провода он подал питание от гальванической батареи, а на выводы второго подключил гальванометр. Конструкция была похожа на современный трансформатор. Периодически включая и выключая напряжение на первом проводе, он наблюдал всплески на гальванометре.

Гальванометр — это высокочувствительный прибор для измерения силы токов малой величины.

Таким образом было изображено влияние магнитного поля, образовавшегося в результате протекания тока в первом проводе, на состояние второго проводника. Это воздействие передавалось от первого ко второму через сердечник – металлический тор. В результате исследований было обнаружено и влияние постоянного магнита, который двигается в катушке, на её обмотку.

Тогда Фарадей объяснял явление электромагнитной индукции с точки зрения силовых линий. Еще одной была установка для генерирования постоянного тока: медный диск вращался вблизи магнита, а скользящий по нему провод был токосъёмником. Это изобретение так и называется — диск Фарадея.

Ученные того периода не признали идеи Фарадея, но Максвелл взял исследования для основы своей магнитной теории. В 1836 г. Майкл Фарадей установил соотношения для электрохимических процессов, которые назвали Законами электролиза Фарадея. Первый описывает соотношения выделенной на электроде массы вещества и протекающего тока, а второй соотношения массы вещества в растворе и выделенного на электроде, для определенного количества электричества.

Электродинамика

Первые работы применяются в физике, конкретно в описании работы электрических машин и аппаратов (трансформаторов, двигателей и пр.). Закон Фарадея гласит:

Для контура индуцированная ЭДС прямо пропорциональна величине скорости магнитного потока, который перемещается через этот контур со знаком минус.

Это можно сказать простыми словами: чем быстрее магнитный поток движется через контур, тем больше на его выводах генерируется ЭДС.

Формула выглядит следующим образом:

Здесь dФ – магнитный поток, а dt – единица времени. Известно, что первая производная по времени – это скорость. Т.е скорость перемещения магнитного потока в данном конкретном случае. Кстати перемещаться может, как и источник магнитного поля (катушка с током – электромагнит, или постоянный магнит), так и контур.

Здесь же поток можно выразить по такой формуле:

B – магнитное поле, а dS – площадь поверхности.

Если рассматривать катушку с плотнонамотанными витками, при этом в количестве витков N, то закон Фарадея выглядит следующим образом:

Магнитный поток в формуле на один виток, измеряется в Веберах. Ток, протекающий в контуре, называется индукционным.

Электромагнитная индукция – явление протекания тока в замкнутом контуре под воздействием внешнего магнитного поля.

В формулах выше вы могли заметить знаки модуля, без них она имеет слегка иной вид, такой как было сказано в первой формулировке, со знаком минус.

Знак минус объясняет правило Ленца. Ток, возникающий в контуре, создает магнитное поле, оно направлено противоположно. Это является следствием закона сохранения энергии.

Направление индукционного тока можно определить по правилу правой руки или , мы его рассматривали на нашем сайте подробно.

Как уже было сказано, благодаря явлению электромагнитной индукции работают электрические машины трансформаторы, генераторы и двигатели. На иллюстрации показано протекание тока в обмотке якоря под воздействием магнитного поля статора. В случае с генератором, при вращении его ротора внешними силами в обмотках ротора возникает ЭДС, ток порождает магнитное поле направленное противоположно (тот самый знак минус в формуле). Чем больше ток, потребляемый нагрузкой генератора, тем больше это магнитное поле, и тем больше затрудняется его вращение.

И наоборот — при протекании тока в роторе возникает поле, которое взаимодействует с полем статора и ротор начинает вращаться. При нагрузке на вал ток в статоре и в роторе повышается, при этом нужно обеспечить переключение обмоток, но это уже другая тема, связанная с устройством электрических машин.

В основе работы трансформатора источником движущегося магнитного потока является переменное магнитное поле, возникающее в следствие протекания в первичной обмотке переменного тока.

Если вы желаете более подробно изучить вопрос, рекомендуем просмотреть видео, на котором легко и доступно рассказывается Закон Фарадея для электромагнитной индукции:

Электролиз

Кроме исследований ЭДС и электромагнитной индукции ученный сделал большие открытия и в других дисциплинах, в том числе химии.

При протекании тока через электролит ионы (положительные и отрицательные) начинают устремляться к электродам. Отрицательные движутся к аноду, положительные к катоду. При этом на одном из электродов выделяется определенная масса вещества, которое содержится в электролите.

Фарадей проводил эксперименты, пропуская разный ток через электролит и измеряя массу вещества отложившегося на электродах, вывел закономерности.

m – масса вещества, q – заряд, а k – зависит от состава электролита.

А заряд можно выразить через ток за промежуток времени:

I=q/t , тогда q = i*t

Теперь можно определить массу вещества, которое выделится, зная ток и время, которое он протекал. Это называется Первый закон электролиза Фарадея.

Второй закон:

Масса химического элемента, который осядет на электроде, прямо пропорциональна эквивалентной массе элемента (молярной массе разделенной на число, которое зависит от химической реакции, в которой участвует вещество).

С учетом вышесказанного эти законы объединяются в формулу:

m – масса вещества, которое выделилось в граммах, n – количество переносимых электронов в электродном процессе, F=986485 Кл/моль – число Фарадея, t – время в секундах, M молярная масса вещества г/моль.

В реальности же из-за разных причин, масса выделяемого вещества меньше чем расчетная (при расчетах с учетом протекающего тока). Отношение теоретической и реальной масс называют выходом по току:

B т = 100% * m расч /m теор

Законы Фарадея внесли существенный вклад в развитие современной науки, благодаря его работам мы имеем электродвигатели и генераторы электроэнергии (а также работам его последователей). Работа ЭДС и явления электромагнитной индукции подарили нам большую часть современного электрооборудования, в том числе и громкоговорители и микрофоны, без которых невозможно прослушивание записей и голосовая связь. Процессы электролиза применяются в гальваническом методе покрытия материалов, что несет как декоративную ценность, так и практическую.

Похожие материалы:

Нравится(0 ) Не нравится(0 )

Явление электромагнитной индукции было открыто Майклом Фарадеем в 1831 г. Он опытным путем установил, что при изменении магнитного по­ля внутри замкнутого контура в нем возникает элек­трический ток, который называютиндукционным током. Опыты Фарадея можно воспроизвести сле­дующим образом: при внесении или вынесении маг­нита в катушку, замкнутую на гальванометр, в ка­тушке возникает индукционный ток (рис. 24). Если рядом расположить две катушки (например, на об­щем сердечнике или одну катушку внутри другой) и одну катушку через ключ соединить с источником тока, то при замыкании или размыкании ключа в цепи первой катушки во второй катушке появится индукционный ток (рис. 25). Объяснение этого явле­ния было дано Максвеллом. Любое переменное маг­нитное поле всегда порождает переменное электриче­ское поле.

Для количественной характеристики процесса изменения магнитного поля через замкнутый контур вводится физическая величина под названием маг­нитный поток.Магнитным потоком через замкну­тый контур площадью S называют физическую вели­чину, равную произведению модуля вектора магнит­ной индукции В на площадь контура S и на косинус угла а между направлением вектора магнитной ин­дукции и нормалью к площади контура. Ф = BS cos α (рис. 26).

Опытным путем был установлен основной за­кон электромагнитной индукции:ЭДС индукции в замкнутом контуре равна по величине скорости из-менения магнитного потока через контур. ξ = ΔФ/t..

Если рассматривать катушку, содержащую п витков, то формула основного закона электромагнитной ин­дукции будет выглядеть так: ξ = n ΔФ/t.

Единица измерения магнитного потока Ф — вебер (Вб): 1В6 =1Β c.

Из основного закона ΔФ =ξ t следует смысл размерности: 1 вебер — это величина такого магнит­ного потока, который, уменьшаясь до нуля за одну секунду, через замкнутый контур наводит в нем ЭДС индукции 1 В.

Классической демонстрацией основного закона электромагнитной индукции является первый опыт Фарадея: чем быстрее перемещать магнит через вит­ки катушки, тем больше возникает индукционный ток в ней, а значит, и ЭДС индукции.

Зависимость направления индукционного тока от характера изменения магнитного поля через замкнутый контур в 1833 г. опытным путем устано­вил русский ученый Ленц. Он сформулировал прави­ло, носящее его имя. Индукционный ток имеет та­кое направление, при котором его магнитное поле стремится скомпенсировать изменение внешнего магнитного потока через контур. Ленцем был скон­струирован прибор, представляющий собой два алю­миниевых кольца, сплошное и разрезанное, укреп­ленные на алюминиевой перекладине и имеющие возможность вращаться вокруг оси, как коромысло. (рис. 27). При внесении магнита в сплошное кольцо оно начинало «убегать» от магнита, поворачивая со­ответственно коромысло. При вынесении магнита из кольца кольцо стремилось «догнать» магнит. При движении магнита внутри разрезанного кольца ни­какого эффекта не происходило. Ленц объяснял опыт тем, что магнитное поле индукционного тока стре­милось компенсировать изменение внешнего магнит­ного потока.

В первой экспериментальной демонстрации электромагнитной индукции (август 1831) Фарадей обмотал двумя проводами противоположные стороны железного тора (конструкция похожа на современный трансформатор). Основываясь на своей оценке недавно обнаруженного свойства электромагнита, он ожидал, что при включении тока в одном проводе особого рода волна пройдёт сквозь тор и вызовет некоторое электрическое влияние на его противоположной стороне. Он подключил один провод к гальванометру и смотрел на него, когда другой провод подключал к батарее. В самом деле, он увидел кратковременный всплеск тока (который он назвал «волной электричества»), когда подключал провод к батарее, и другой такой же всплеск, когда отключал его. В течение двух месяцев Фарадей нашёл несколько других проявлений электромагнитной индукции. Например, он увидел всплески тока, когда быстро вставлял магнит в катушку и вытаскивал его обратно, он генерировал постоянный ток во вращающемся вблизи магнита медном диске со скользящим электрическим проводом («диск Фарадея ») .

Фарадей объяснил электромагнитную индукцию с использованием концепции так называемых силовых линий . Однако, большинство учёных того времени отклонили его теоретические идеи, в основном потому, что они не были сформулированы математически. Исключение составил Максвелл , который использовал идеи Фарадея в качестве основы для своей количественной электромагнитной теории. В работах Максвелла аспект изменения во времени электромагнитной индукции выражен в виде дифференциальных уравнений. Оливер Хевисайд назвал это законом Фарадея, хотя он несколько отличается по форме от первоначального варианта закона Фарадея и не учитывает индуцирование ЭДС при движении. Версия Хевисайда является формой признанной сегодня группы уравнений, известных как уравнения Максвелла .

Закон Фарадея как два различных явления

Некоторые физики отмечают, что закон Фарадея в одном уравнении описывает два разных явления: двигательную ЭДС , генерируемую действием магнитной силы на движущийся провод, и трансформаторную ЭДС , генерируемую действием электрической силы вследствие изменения магнитного поля. Джеймс Клерк Максвелл обратил внимание на этот факт в своей работе О физических силовых линиях в 1861 году. Во второй половине части II этого труда Максвелл даёт отдельное физическое объяснение для каждого из этих двух явлений. Ссылка на эти два аспекта электромагнитной индукции имеется в некоторых современных учебниках. Как пишет Ричард Фейнман:

Таким образом, «правило потока» о том, что ЭДС в цепи равна скорости изменения магнитного потока через контур, применяется независимо от причины изменения потока: то ли потому что поле изменяется, то ли потому что цепь движется (или и то, и другое)…. В нашем объяснении правила мы использовали два совершенно различных закона для двух случаев  –    v × B {\displaystyle {\stackrel {\mathbf {v\times B} }{}}}   для «движущейся цепи» и   ∇ x E = − ∂ t B {\displaystyle {\stackrel {\mathbf {\nabla \ x\ E\ =\ -\partial _{\ t}B} }{}}}   для «меняющегося поля».

Мы не знаем никакого аналогичного положения в физике, когда такие простые и точные общие принципы требовали бы для своего реального понимания анализа с точки зрения двух различных явлений.

Отражение этой очевидной дихотомии было одним из основных путей, которые привели Эйнштейна к разработке специальной теории относительности :

Известно, что электродинамика Максвелла — как её обычно понимают в настоящее время — при применении к движущимся телам приводит к асимметрии, которая, как кажется, не присуща этому явлению. Возьмем, к примеру, электродинамическое взаимодействие магнита и проводника. Наблюдаемое явление зависит только от относительного движения проводника и магнита, тогда как обычное мнение рисует резкое различие между этими двумя случаями, в которых либо одно, либо другое тело находится в движении. Ибо, если магнит находится в движении, а проводник покоится, в окрестности магнита возникает электрическое поле с определенной плотностью энергии, создавая ток там, где расположен проводник. Но если магнит покоится, а проводник движется, то в окрестности магнита никакое электрическое поле не возникает. В проводнике, однако, мы находим электродвижущую силу, для которой не существует соответствующей энергии самой по себе, но которая вызывает — предполагая равенство относительного движения в двух обсуждаемых случаях — электрические токи по тому же направлению и той же интенсивности, как в первом случае.

Примеры подобного рода вместе с неудачной попыткой обнаружить какое-либо движение Земли относительно «светоносной среды» предполагают, что явления электродинамики, а также механики не обладают свойствами, соответствующими идее абсолютного покоя.

Альберт Эйнштейн , К электродинамике движущихся тел

Поток через поверхность и ЭДС в контуре

Закон электромагнитной индукции Фарадея использует понятие магнитного потока Φ B через замкнутую поверхность Σ, который определён через поверхностный интеграл :

Φ = ∬ S B n ⋅ d S , {\displaystyle \Phi =\iint \limits _{S}\mathbf {B_{n}} \cdot d\mathbf {S} ,}

где dS — площадь элемента поверхности Σ(t ), B — магнитное поле, а B ·d S — скалярное произведение B и d S . Предполагается, что поверхность имеет «устье», очерченное замкнутой кривой, обозначенной ∂Σ(t ). Закон индукции Фарадея утверждает, что когда поток изменяется, то при перемещении единичного положительного пробного заряда по замкнутой кривой ∂Σ совершается работа E {\displaystyle {\mathcal {E}}} , величина которой определяется по формуле:

| E | = | d Φ d t | , {\displaystyle |{\mathcal {E}}|=\left|{{d\Phi } \over dt}\right|\ ,}

где | E | {\displaystyle |{\mathcal {E}}|} — величина электродвижущей силы (ЭДС) в вольтах , а Φ B — магнитный поток в веберах . Направление электродвижущей силы определяется законом Ленца .

На рис. 4 показан шпиндель, образованный двумя дисками с проводящими ободами, и проводники, расположенные вертикально между этими ободами. ток скользящими контактами подается на проводящие обода. Эта конструкция вращается в магнитном поле, которое направлено радиально наружу и имеет одно и то же значение в любом направлении. т.е. мгновенная скорость проводников, ток в них и магнитная индукция, образуют правую тройку, что заставляет проводники вращаться.

Сила Лоренца

В этом случае на проводники действует Сила Ампера а на единичный заряд в проводнике Сила Лоренца — поток вектора магнитной индукции B , ток в проводниках, соединяющие проводящие обода, направлен нормально к вектору магнитной индукции, тогда сила действующая на заряд в проводнике будет равна

F = q B v . {\displaystyle F=qBv\,.}

где v = скорости движущегося заряда

Следовательно, сила действующая на проводники

F = I B ℓ , {\displaystyle {\mathcal {F}}=IB\ell ,}

где l длина проводников

Здесь мы использовали B как некую данность, на самом деле она зависит от геометрических размеров ободов конструкции и это значение можно вычислить используя Закон Био - Савара - Лапласа . Данный эффект используется и в другом устройстве называемом Рельсотрон

Закон Фарадея

Интуитивно привлекательный, но ошибочный подход к использованию правила потока выражает поток через цепь по формуле Φ B = B w ℓ, где w — ширина движущейся петли.

Ошибочность такого подхода в том что это не рамка в обычном понимании этого слова. прямоугольник на рисунке образован отдельными проводниками, замкнутыми на обод. Как видно на рисунке ток по обоим проводника течет в одном направлении, т.е. здесь отсутствует понятие «замкнутый контур»

Наиболее простое и понятное объяснение этому эффекту дает понятие сила Ампера . Т.е. вертикальный проводник может быть вообще один, чтобы не вводить в заблуждение. Или же проводник конечной толщины может быть расположен на оси соединяющие обода. Диаметр проводника должен быть конечным и отличатся от нуля чтобы момент силы Ампера был не нулевой.

Уравнение Фарадея — Максвелла

Переменное магнитное поле создаёт электрическое поле, описываемое уравнением Фарадея — Максвелла:

∇ × E = − ∂ B ∂ t {\displaystyle \nabla \times \mathbf {E} =-{\frac {\partial \mathbf {B} }{\partial t}}}

∇ × {\displaystyle \nabla \times } обозначает ротор E — электрическое поле B — плотность магнитного потока .

Это уравнение присутствует в современной системе уравнений Максвелла , часто его называют законом Фарадея. Однако, поскольку оно содержит только частные производные по времени, его применение ограничено ситуациями, когда заряд покоится в переменном по времени магнитном поле. Оно не учитывает [ ] электромагнитную индукцию в случаях, когда заряженная частица движется в магнитном поле.

В другом виде закон Фарадея может быть записан через интегральную форму теоремы Кельвина-Стокса :

∮ ∂ Σ ⁡ E ⋅ d ℓ = − ∫ Σ ∂ ∂ t B ⋅ d A {\displaystyle \oint _{\partial \Sigma }\mathbf {E} \cdot d{\boldsymbol {\ell }}=-\int _{\Sigma }{\partial \over {\partial t}}\mathbf {B} \cdot d\mathbf {A} }

Для выполнения интегрирования требуется независимая от времени поверхность Σ (рассматриваемая в данном контексте как часть интерпретации частных производных). Как показано на рис. 6:

Σ — поверхность, ограниченная замкнутым контуром ∂Σ , причём, как Σ , так и ∂Σ являются фиксированными, не зависящими от времени, E — электрическое поле, d — бесконечно малый элемент контура ∂Σ , B — магнитное поле , dA — бесконечно малый элемент вектора поверхности Σ .

Элементы d и dA имеют неопределённые знаки. Чтобы установить правильные знаки, используется правило правой руки , как описано в статье о теореме Кельвина-Стокса . Для плоской поверхности Σ положительное направление элемента пути d кривой ∂Σ определяется правилом правой руки, по которому на это направление указывают четыре пальца правой руки, когда большой палец указывает в направлении нормали n к поверхности Σ.

Интеграл по ∂Σ называется интеграл по пути или криволинейным интегралом . Поверхностный интеграл в правой части уравнения Фарадея-Максвелла является явным выражением для магнитного потока Φ B через Σ . Обратите внимание, что ненулевой интеграл по пути для E отличается от поведения электрического поля, создаваемого зарядами. Генерируемое зарядом E -поле может быть выражено как градиент скалярного поля , которое является решением уравнения Пуассона и имеет нулевой интеграл по пути.

Интегральное уравнение справедливо для любого пути ∂Σ в пространстве и любой поверхности Σ , для которой этот путь является границей.

D d t ∫ A B d A = ∫ A (∂ B ∂ t + v div B + rot (B × v)) d A {\displaystyle {\frac {\text{d}}{{\text{d}}t}}\int \limits _{A}{\mathbf {B} }{\text{ d}}\mathbf {A} =\int \limits _{A}{\left({\frac {\partial \mathbf {B} }{\partial t}}+\mathbf {v} \ {\text{div}}\ \mathbf {B} +{\text{rot}}\;(\mathbf {B} \times \mathbf {v})\right)\;{\text{d}}}\mathbf {A} }

и принимая во внимание div B = 0 {\displaystyle {\text{div}}\mathbf {B} =0} (Ряд Гаусса), B × v = − v × B {\displaystyle \mathbf {B} \times \mathbf {v} =-\mathbf {v} \times \mathbf {B} } (Векторное произведение) и ∫ A rot X d A = ∮ ∂ A ⁡ X d ℓ {\displaystyle \int _{A}{\text{rot}}\;\mathbf {X} \;\mathrm {d} \mathbf {A} =\oint _{\partial A}\mathbf {X} \;{\text{d}}{\boldsymbol {\ell }}} (теорема Кельвина - Стокса), мы находим, что полная производная магнитного потока может быть выражена

∫ Σ ∂ B ∂ t d A = d d t ∫ Σ B d A + ∮ ∂ Σ ⁡ v × B d ℓ {\displaystyle \int \limits _{\Sigma }{\frac {\partial \mathbf {B} }{\partial t}}{\textrm {d}}\mathbf {A} ={\frac {\text{d}}{{\text{d}}t}}\int \limits _{\Sigma }{\mathbf {B} }{\text{ d}}\mathbf {A} +\oint _{\partial \Sigma }\mathbf {v} \times \mathbf {B} \,{\text{d}}{\boldsymbol {\ell }}}

Добавляя член ∮ ⁡ v × B d ℓ {\displaystyle \oint \mathbf {v} \times \mathbf {B} \mathrm {d} \mathbf {\ell } } к обеим частям уравнения Фарадея-Максвелла и вводя вышеприведённое уравнение, мы получаем:

∮ ∂ Σ ⁡ (E + v × B) d ℓ = − ∫ Σ ∂ ∂ t B d A ⏟ induced emf + ∮ ∂ Σ ⁡ v × B d ℓ ⏟ motional emf = − d d t ∫ Σ B d A , {\displaystyle \oint \limits _{\partial \Sigma }{(\mathbf {E} +\mathbf {v} \times \mathbf {B})}{\text{d}}\ell =\underbrace {-\int \limits _{\Sigma }{\frac {\partial }{\partial t}}\mathbf {B} {\text{d}}\mathbf {A} } _{{\text{induced}}\ {\text{emf}}}+\underbrace {\oint \limits _{\partial \Sigma }{\mathbf {v} }\times \mathbf {B} {\text{d}}\ell } _{{\text{motional}}\ {\text{emf}}}=-{\frac {\text{d}}{{\text{d}}t}}\int \limits _{\Sigma }{\mathbf {B} }{\text{ d}}\mathbf {A} ,}

что и является законом Фарадея. Таким образом, закон Фарадея и уравнения Фарадея-Максвелла физически эквивалентны.

Рис. 7 показывает интерпретацию вклада магнитной силы в ЭДС в левой части уравнения. Площадь, заметаемая сегментом d кривой ∂Σ за время dt при движении со скоростью v , равна:

d A = − d ℓ × v d t , {\displaystyle d\mathbf {A} =-d{\boldsymbol {\ell \times v}}dt\ ,}

так что изменение магнитного потока ΔΦ B через часть поверхности, ограниченной ∂Σ за время dt , равно:

d Δ Φ B d t = − B ⋅ d ℓ × v = − v × B ⋅ d ℓ , {\displaystyle {\frac {d\Delta \Phi _{B}}{dt}}=-\mathbf {B} \cdot \ d{\boldsymbol {\ell \times v}}\ =-\mathbf {v} \times \mathbf {B} \cdot \ d{\boldsymbol {\ell }}\ ,}

и если сложить эти ΔΦ B -вклады вокруг петли для всех сегментов d , мы получим суммарный вклад магнитной силы в закон Фарадея. То есть этот термин связан с двигательной ЭДС.

Пример 3: точка зрения движущегося наблюдателя

Возвращаясь к примеру на рис. 3, в движущейся системе отсчета выявляется тесная связь между E — и B -полями, а также между двигательной и индуцированной ЭДС. Представьте себе наблюдателя, движущегося вместе с петлёй. Наблюдатель вычисляет ЭДС в петле с использованием как закона Лоренца, так и с использованием закона электромагнитной индукции Фарадея. Поскольку этот наблюдатель движется с петлей, он не видит никакого движения петли, то есть нулевую величину v × B . Однако, поскольку поле B меняется в точке x , движущийся наблюдатель видит изменяющееся во времени магнитного поля, а именно:

B = k B (x + v t) , {\displaystyle \mathbf {B} =\mathbf {k} {B}(x+vt)\ ,}

где k — единичный вектор в направлении z .

Закон Лоренца

Уравнение Фарадея-Максвелла говорит, что движущийся наблюдатель видит электрическое поле E y в направлении оси y , определяемое по формуле:

∇ × E = k d E y d x {\displaystyle \nabla \times \mathbf {E} =\mathbf {k} \ {\frac {dE_{y}}{dx}}} = − ∂ B ∂ t = − k d B (x + v t) d t = − k d B d x v , {\displaystyle =-{\frac {\partial \mathbf {B} }{\partial t}}=-\mathbf {k} {\frac {dB(x+vt)}{dt}}=-\mathbf {k} {\frac {dB}{dx}}v\ \ ,} d B d t = d B d (x + v t) d (x + v t) d t = d B d x v . {\displaystyle {\frac {dB}{dt}}={\frac {dB}{d(x+vt)}}{\frac {d(x+vt)}{dt}}={\frac {dB}{dx}}v\ .}

Решение для E y с точностью до постоянной, которая ничего не добавляет в интеграл по петле:

E y (x , t) = − B (x + v t) v . {\displaystyle E_{y}(x,\ t)=-B(x+vt)\ v\ .}

Используя закон Лоренца, в котором имеется только компонента электрического поля, наблюдатель может вычислить ЭДС по петле за время t по формуле:

E = − ℓ [ E y (x C + w / 2 , t) − E y (x C − w / 2 , t) ] {\displaystyle {\mathcal {E}}=-\ell } = v ℓ [ B (x C + w / 2 + v t) − B (x C − w / 2 + v t) ] , {\displaystyle =v\ell \ ,}

и мы видим, что точно такой же результат найден для неподвижного наблюдателя, который видит, что центр масс x C сдвинулся на величину x C + v t . Однако, движущийся наблюдатель получил результат под впечатлением, что в законе Лоренца действовала только электрическая составляющая, тогда как неподвижный наблюдатель думал, что действовала только магнитная составляющая.{x_{C}+w/2}{\frac {d}{dx}}B(x+vt)\ v\ dx} = v ℓ [ B (x C + w / 2 + v t) − B (x C − w / 2 + v t) ] , {\displaystyle =v\ell \ \ ,}

и мы видим тот же результат. Производная по времени используется при интегрировании, поскольку пределы интегрирования не зависят от времени. Опять же, для преобразования производной по времени в производную по x используются методы дифференцирования сложной функции.

Неподвижный наблюдатель видит ЭДС как двигательную , тогда как движущийся наблюдатель думает, что это индуцированная ЭДС.

Электрический генератор

Явление возникновения ЭДС, порождённой по закону индукции Фарадея из-за относительного движения контура и магнитного поля, лежит в основе работы электрических генераторов . Если постоянный магнит перемещается относительно проводника или наоборот, проводник перемещается относительно магнита, то возникает электродвижущая сила. Если проводник подключён к электрической нагрузке, то через неё будет течь ток, и следовательно, механическая энергия движения будет превращаться в электрическую энергию. Например, дисковый генератор построен по тому же принципу, как изображено на рис. 4. Другой реализацией этой идеи является диск Фарадея , показанный в упрощённом виде на рис. 8. Обратите внимание, что и анализ рис. 5, и прямое применение закона силы Лоренца показывают, что твёрдый проводящий диск работает одинаковым образом.

В примере диска Фарадея диск вращается в однородном магнитном поле, перпендикулярном диску, в результате чего возникает ток в радиальном плече благодаря силе Лоренца. Интересно понять, как получается, что чтобы управлять этим током, необходима механическая работа. Когда генерируемый ток течёт через проводящий обод, по закону Ампера этот ток создаёт магнитное поле (на рис. 8 оно подписано «индуцированное B» — Induced B). Обод, таким образом, становится электромагнитом , который сопротивляется вращению диска (пример правила Ленца). В дальней части рисунка обратный ток течёт от вращающегося плеча через дальнюю сторону обода к нижней щётке. Поле В, создаваемое этим обратным током, противоположно приложенному полю, вызывая сокращение потока через дальнюю сторону цепи, в противовес увеличению потока, вызванного вращением. На ближней стороне рисунка обратный ток течёт от вращающегося плеча через ближнюю сторону обода к нижней щётке. Индуцированное поле B увеличивает поток по эту сторону цепи, в противовес снижению потока, вызванного вращением. Таким образом, обе стороны цепи генерируют ЭДС, препятствующую вращению. Энергия, необходимая для поддержания движения диска в противовес этой реактивной силе, в точности равна вырабатываемой электрической энергии (плюс энергия на компенсацию потерь из-за трения, из-за выделения тепла Джоуля и прочее). Такое поведение является общим для всех генераторов преобразования механической энергии в электрическую.

Хотя закон Фарадея описывает работу любых электрических генераторов, детальный механизм в разных случаях может отличаться. Когда магнит вращается вокруг неподвижного проводника, меняющееся магнитное поле создаёт электрическое поле, как описано в уравнении Максвелла-Фарадея, и это электрическое поле толкает заряды через проводник. Этот случай называется индуцированной ЭДС. С другой стороны, когда магнит неподвижен, а проводник вращается, на движущиеся заряды воздействует магнитная сила (как описывается законом Лоренца), и эта магнитная сила толкает заряды через проводник. Этот случай называется двигательной ЭДС.

Электродвигатель

Электрический генератор может работать в «обратном направлении» и становиться двигателем. Рассмотрим, например, диск Фарадея. Предположим, постоянный ток течёт через проводящее радиальное плечо от какого-либо напряжения. Тогда по закону силы Лоренца на этот движущийся заряд воздействует сила в магнитном поле B , которая будет вращать диск в направлении, определённым правилом левой руки. При отсутствии эффектов, вызывающих диссипативные потери, таких как трение или тепло Джоуля , диск будет вращаться с такой скоростью, чтобы d Φ B / dt было равно напряжению, вызывающему ток.

Электрический трансформатор

ЭДС, предсказанная законом Фарадея, является также причиной работы электрических трансформаторов. Когда электрический ток в проволочной петле изменяется, меняющийся ток создаёт переменное магнитное поле. Второй провод в доступном для него магнитном поле будет испытывать эти изменения магнитного поля как изменения связанного с ним магнитного потока d Φ B / d t . Электродвижущая сила, возникающая во второй петле, называется индуцированной ЭДС или ЭДС трансформатора . Если два конца этой петли связать через электрическую нагрузку, то через неё потечёт ток.

В 1831 году мир впервые узнал о понятии электромагнитной индукции. Именно тогда Майкл Фарадей обнаружил это явление, ставшее в итоге важнейшим открытием в электродинамике.

История развития и опыты Фарадея

До середины XIX века считалось, что электрическое и магнитное поле не имеют никакой связи, и природа их существования различна. Но М. Фарадей был уверен в единой природе этих полей и их свойств. Явление электромагнитной индукции, обнаруженное им, впоследствии стало фундаментом для устройства генераторов всех электростанций. Благодаря этому открытию знания человечества о электромагнетизме шагнули далеко вперед.

Фарадей проделал следующий опыт: он замыкал цепь в катушке I и вокруг нее возрастало магнитное поле. Далее линии индукции данного магнитного поля пересекали катушку II, в которой возникал индукционный ток.

Рис. 1. Схема опыта Фарадея

На самом деле, одновременно с Фарадеем, но независимо от него, другой ученый Джозеф Генри обнаружил это явление. Однако Фарадей опубликовал свои исследования раньше. Таким образом, автором закона электромагнитной индукции стал Майкл Фарадей.

Сколько бы экспериментов не проводил Фарадей, неизменным оставалось одно условие: для образования индукционного тока важным является изменение магнитного потока, пронизывающего замкнутый проводящий контур (катушку).

Закон Фарадея

Явление электромагнитной индукции определяется возникновением электрического тока в замкнутом электропроводящем контуре при изменении магнитного потока через площадь этого контура.

Основной закон Фарадея заключается в том, что электродвижущая сила (ЭДС) прямо пропорциональна скорости изменения магнитного потока.

Формула закона электромагнитной индукции Фарадея выглядит следующим образом:

Рис. 2. Формула закона электромагнитной индукции

И если сама формула, исходя из вышесказанных объяснений не порождает вопросов, то знак «-» может вызвать сомнения. Оказывается существует правило Ленца – русского ученого, который проводил свои исследования, основываясь на постулатах Фарадея. По Ленцу знак «-» указывает на направление возникающей ЭДС, т.е. индукционный ток направлен так, что магнитный поток, который он создает, через площадь, ограниченную контуром, стремится препятствовать тому изменению потока, которое вызывает данный ток.

Закон Фарадея-Максвелла

В 1873 Дж.К.Максвелл по-новому изложил теорию электромагнитного поля. Уравнения, которые он вывел, легли в основу современной радиотехники и электротехники. Они выражаются следующим образом:

  • Edl = -dФ/dt – уравнение электродвижущей силы
  • Hdl = -dN/dt – уравнение магнитодвижущей силы.

Где E – напряженность электрического поля на участке dl; H – напряженность магнитного поля на участке dl; N – поток электрической индукции, t – время.

Симметричный характер данных уравнений устанавливает связь электрических и магнитных явлений, а также магнитных с электрическими. физический смысл, которым определяются эти уравнения, можно выразить следующими положениями:

  • если электрическое поле изменяется, то это изменение всегда сопровождается магнитным полем.
  • если магнитное поле изменяется, то это изменение всегда сопровождается электрическим полем.

Рис. 3. Возникновение вихревого магнитного поля

Также Максвелл установил, что распространение электромагнитного поля равна скорости распространения света.

Всего получено оценок: 134.

Закон электромагнитной индукции. Правило Ленца и Фарадея

Сегодня мы раскроем такой феномен физики, как «закон электромагнитной индукции». Расскажем, почему Фарадей провел опыты, приведем формулу и объясним важность явления для повседневной жизни.

Древние боги и физика

Древние люди поклонялись неведомому. И сейчас человека страшит пучина моря и даль космоса. Но наука может объяснить, почему. Субмарины снимают невероятную жизнь океанов на глубине свыше километра, космические телескопы изучают объекты, которые существовали всего лишь через считанные миллионы лет после большого взрыва.

Но тогда люди обожествляли все, что их завораживало и тревожило:

  • восход солнца;
  • пробуждение растений весной;
  • дождь;
  • рождение и смерть.

В каждом предмете и явлении жили неведомые силы, которые управляли миром. До сих пор дети склонны очеловечивать мебель и игрушки. Оставаясь без присмотра взрослых, они фантазируют: одеяло обнимет, табуретка подойдет, окно откроется само по себе.

Пожалуй, первым эволюционным шагом человечества стало умение поддерживать огонь. Антропологи предполагают, что самые ранние костры зажглись от дерева, в которое ударила молния.

Таким образом, электричество сыграло в жизни человечества огромную роль. Первая молния дала толчок к развитию культуры, основной закон электромагнитной индукции привел человечество к современному состоянию.

От уксуса до ядерного реактора

В пирамиде Хеопса были найдены странные керамические сосуды: горлышко запечатано воском, в глубине скрыт металлический цилиндр. На внутренней стороне стенок обнаружили остатки уксуса или кислого вина. Ученые пришли к сенсационному выводу: этот артефакт – батарейка, источник электричества.

Но до 1600 года изучать этот феномен никто не брался. До движущихся электронов исследовали природу статического электричества. О том, что янтарь дает разряды, если его потереть о мех, знали еще древние греки. Цвет этого камня напоминал им свет звезды Электры из Плеяд. А название минерала стало, в свою очередь, поводом окрестить физическое явление.

Первый примитивный источник постоянного тока был построен в 1800 году

Естественно, как только появился достаточно мощный конденсатор, ученые принялись изучать свойства подключенного к нему проводника. В 1820 году датский ученый Ханс Кристиан Эрстед обнаружил, что магнитная стрелка отклоняется рядом с включенным в сеть проводником. Данный факт дал толчок к открытию закона электромагнитной индукции Фарадеем (формула будет приведена чуть ниже), который позволил человечеству добывать электричество из воды, ветра и ядерного топлива.

Примитивное, но современное

Физическая основа опытов Макса Фарадея была заложена Эрстедом. Если включенный проводник влияет на магнит, то верно и обратное: намагниченный проводник должен вызывать ток.

Структура опыта, который помог вывести закон электромагнитной индукции (ЭДС как понятие мы рассмотрим чуть позже), была весьма проста. Смотанную в пружину проволоку подключили к прибору, который регистрирует ток. К виткам ученый поднес большой магнит. Пока магнит двигался рядом с контуром, прибор регистрировал поток электронов.

С тех пор техника усовершенствовалась, но основной принцип создания электричества на огромных станциях пока что тот же: движущийся магнит возбуждает ток в смотанном пружиной проводнике.

Развитие идеи

Самый первый опыт убедил Фарадея, что электрическое и магнитное поля взаимосвязаны. Но требовалось выяснить, как именно. Возникает ли вокруг проводника с током еще и магнитное поле или они просто способны влиять друг на друга? Поэтому ученый пошел дальше. Он смотал одну проволоку, подвел к ней ток, и эту катушку вдвинул в другую пружину. И тоже получил электричество. Этот опыт доказал, что движущиеся электроны создают не только электрическое, но и магнитное поле. Позже ученые выяснили, как они располагаются в пространстве относительно друг друга. Электромагнитное поле – это и та причина, по которой существует свет.

Экспериментируя с разными вариантами взаимодействия проводников под напряжением, Фарадей выяснил: ток передается лучше всего, если и первую, и вторую катушки намотать на один общий металлический сердечник. Формула, выражающая закон электромагнитной индукции, была выведена именно на этом приборе.

Формула и ее составляющие

Теперь, когда история изучения электричества доведена до эксперимента Фарадея, пора написать формулу:

ε = -dΦ / dt.

Расшифруем:

ε – это электродвижущая сила (сокращенно ЭДС). В зависимости от величины ε электроны перемещаются в проводнике интенсивнее или слабее. На ЭДС влияет мощность источника, а на нее – напряженность электромагнитного поля.

Φ – величина магнитного потока, который проходит в данный момент через заданную площадь. Фарадей сворачивал проволоку в пружину, так как ему требовалась определенное пространство, сквозь которое проходил бы проводник. Конечно, можно было бы изготовить очень толстый проводник, но это было бы дорого. Форму круга ученый выбрал потому, что у этой плоской фигуры соотношение площади к длине поверхности наибольшее. Это самая энергетически эффективная форма. Поэтому капли воды на плоской поверхности становятся круглыми. К тому же пружину с круглым сечением гораздо проще получить: достаточно лишь намотать проволоку на какой-то круглый предмет.

t – время, за которое поток прошел сквозь контур.

Приставка d в формуле закона электромагнитной индукции означает, что величина дифференциальная. То есть маленький магнитный поток надо продифференцировать по небольшим отрезкам времени, чтобы получить конечный результат. Это математическое действие требует от людей некоторой подготовленности. Чтобы лучше понять формулу, мы настоятельно рекомендуем читателю вспомнить дифференцирование и интегрирование.

Следствия из закона

Сразу после открытия Фарадея физики стали исследовать явление электромагнитной индукции. Закон Ленца, например, был выведен экспериментально российским ученым. Именно это правило добавило минус в конечную формулу.

Вид у него такой: направление индукционного тока не случайно; поток электронов во второй обмотке как бы стремится уменьшить действие тока в первой обмотке. То есть возникновение электромагнитной индукции – это фактически сопротивление второй пружины вмешательству в «личную жизнь».

Правило Ленца имеет и другое следствие.

  • если ток в первой катушке будет возрастать, то ток второй пружины тоже будет стремиться к увеличению;
  • если ток в индуцирующей обмотке будет падать, то уменьшится и ток во второй.

Согласно этому правилу, проводник, в котором возникает индуцированный ток, фактически стремится скомпенсировать действие изменяющегося магнитного потока.

Зерно и осел

Использовать простейшие механизмы себе на благо люди стремились давно. Помол муки – дело сложное. Некоторые племена растирают зерно вручную: кладут пшеницу на один камень, накрывают другим плоским и круглым камнем, и вертят жернов. Но если надо смолоть муку на целую деревню, то одним мускульным трудом не обойтись. Сначала люди догадались привязать к жернову тягловое животное. Ослик тянул за веревку – камень вращался. Потом, вероятно, люди подумали: «Река течет все время, она толкает всякие предметы вниз по течению. Почему бы нам не использовать это на благо?» Так появились водяные мельницы.

Колесо, вода, ветер

Конечно, первые инженеры, которые строили эти сооружения, ничего не знали ни о силе тяготения, из-за которой вода стремится всегда вниз, ни о силе трения или поверхностного натяжения. Но они видели: если поставить в ручей или речку колесо с лопастями на диаметре, то оно не только будет вращаться, но и сможет делать полезную работу.

Но и этот механизм был ограничен: не везде есть проточная вода с достаточно силой течения. Поэтому люди пошли дальше. Они построили мельницы, которые работали от ветра.

Уголь, мазут, бензин

Когда ученые поняли принцип возбуждения электричества, была поставлена техническая задача: получать его в промышленных масштабах. На тот момент (середина девятнадцатого века) мир был охвачен лихорадкой машин. Всю сложную работу стремились поручить расширяющемуся пару.

Но тогда нагреть большие объемы воды умели только ископаемым топливом – углем и мазутом. Поэтому те регионы мира, которые были богаты древними углеродами, сразу привлекли внимание инвесторов и рабочих. А перераспределение людей привело к промышленной революции.

Голландия и Техас

Однако такое положение вещей плохо отразилось на экологии. И ученые задумались: как получать энергию, не разрушая природу? Выручило хорошо забытое старое. Мельница использовала крутящий момент для совершения непосредственно грубой механической работы. Турбины гидроэлектростанций вращают магниты.

На данный момент самое чистое электричество получают из энергии ветра. Инженеры, которые строили первые генераторы Техаса, опирались на опыт ветряных мельниц Голландии.

Закон Фарадея и его вывод из закона сохранения энергии

| на главную | доп. материалы | физика как наука и предмет | электричество и электромагнетизм |

Организационные, контрольно-распорядительные и инженерно-технические услуги
в сфере жилой, коммерческой и иной недвижимости. Московский регион. Официально.

Обобщая результаты своих многочисленных опытов, Фарадей пришел к количественному закону электромагнитной индукции. Он показал, что всякий раз, когда происходит изменение сцепленного с контуром потока магнитной индукции, в контуре возникает индукционный ток; возникновение индукционного тока указывает на наличие в цепи электродвижущей силы, называемой электродвижущей силой электромагнитной индукции. Значение индукционного тока, а следовательно, и э.д.с. электро­магнитной индукции определяются только скоростью изменения магнитного потока, т. е.

 

Теперь необходимо выяснить знак . В § 120 было показано, что знак магнитного потока зависит от выбора положительной нормали к контуру. В свою очередь, положительное направление нормали определяется правилом правого винта. Следовательно, выбирая положительное направление нормали, мы определяем как знак потока магнитной индукции, так и направление тока и э.д.с. в контуре. Пользуясь этими представлениями и выводами, можно соответственно прийти к формулировке закона электромагнитной индукции Фарадея: какова бы ни была причина изменения потока магнитной индукции, охватываемого замкнутым проводящим контуром, возникающая в контуре э. д. с.

                                                   (123.2)

Знак минус показывает, что увеличение потока  вызывает э. д. с. т. е. поле индукционного тока направлено навстречу потоку; уменьшение потока  вызывает т.е. направления потока и поля индукционного тока совпадают. Знак минус в формуле (123.2) определяется правилом Ленца — общим правилом для нахождения направления индукционного тока, выведенного в 1833 г.

Правило Ленца: индукционный ток в контуре имеет всегда такое направление, что создаваемое им магнитное поле препятствует изменению магнитного потока, вызва­вшему этот индукционный ток.

Закон Фарадея (см. (123.2)) может быть непосредственно получен из закона со­хранения энергии, как это впервые сделал Г. Гельмгольц. Рассмотрим проводник с током I, который помещен в однородное магнитное поле, перпендикулярное плоско­сти контура, и может свободно перемещаться (см. рис. 177). Под действием силы Ампера F, направление которой показано на рисунке, проводник перемещается на отрезок dx. Таким образом, сила Ампера производит работу (см. (121.1)) dA=IdФ, где dФ — пересеченный проводником магнитный поток.

Согласно закону сохранения энергии, работа источника тока за время dt () будет складываться из работы на джоулеву теплоту (I2Rdt) и работы по перемещению проводника в магнитном поле (IdФ):

где R — полное сопротивление контура. Тогда

=  есть не что иное, как закон Фарадея (см. (123.2)).

Закон Фарадея можно сформулировать еще таким образом: э.д.с.  электромагнитной индукции в контуре численно равна и противоположна по знаку скорости изменения магнитного потока сквозь поверхность, ограниченную этим контуром. Этот закон является универсальным: э. д. с. не зависит от способа изменения магнитного потока. Э.д.с. электромагнитной индукции выражается в вольтах. Действительно, учитывая, что единицей магнитного потока является вебер (Вб), получим

Какова природа э.д.с. электромагнитной индукции? Если проводник (подвижная перемычка контура на рис. 177) движется в постоянном магнитном поле, то сила Лоренца, действующая на заряды внутри проводника, движущиеся вместе с проводником, будет направлена противоположно току, т. е. она будет создавать в проводнике индукционный ток противоположного направления (за направление электрического тока принимается движение положительных зарядов). Таким образом, возбуждение э.д.с. индукции при движения контура в постоянном магнитном поле объясняется действием силы Лоренца, возникающей при движении проводника.

Согласно закону Фарадея, возникновение э.д.с. электромагнитной индукции воз­можно и в случае неподвижного контура, находящегося в переменном магнитном поле. Однако сила Лоренца на неподвижные заряды не действует, поэтому в данном случае ею нельзя объяснить возникновение э.д.с. индукции. Максвелл для объяснения э.д.с. индукции в неподвижных проводниках предположил, что всякое переменное магнитное поле возбуждает в окружающем пространстве электрическое поле, которое и является причиной возникновения индукционного тока в проводнике. Циркуляция вектора ЕB этого поля по любому неподвижному контуру L проводника представляет собой э. д. с. электромагнитной индукции:

                                                          (123.3)


Закон электромагнитной индукции Фарадея; Правило Ленца; явление самоиндукции; индуктивность; энергия электрического поля

Ответ: Электромагнитная индукция. Магнитный поток. Закон электромагнитной индукции. Правило Ленца.

Явление электромагнитной индукции было открыто Майклом Фарадеем в 1831 г. Он опытным путем установил, что при изменении магнитного поля внутри замкнутого контура в нем возникает электрический ток, который называют индукционным током. Опыты Фарадея можно воспроизвести следующим образом: при внесении или вынесении магнита в катушку, замкнутую на гальванометр, в катушке возникает индукционный ток (рис. 34). Если рядом расположить две катушки (например, на общем сердечнике или одну катушку внутри другой) и одну катушку через ключ соединить с источником тока, то при замыкании или размыкании ключа в цепи первой катушки во второй катушке появится индукционный ток (рис. 35). Объяснение этого явления было дано Максвеллом. Любое переменное магнитное поле всегда порождает переменное электрическое поле.

Для количественной характеристики процесса изменения магнитного поля через замкнутый контур вводится физическая величина под названием «магнитный поток». Магнитным потоком через замкнутый контур площадью S называют физическую величину, равную произведению модуля вектора магнитной индукции Б на площадь контура S и на косинус угла а между направлением вектора магнитной индукции и нормалью к площади контура.

Ф = BS cos а
Опытным путем был установлен основной закон электромагнитной индукции: ЭДС индукции в замкнутом контуре равна по величине скорости изменения магнитного потока через контур. . Если рассматривать катушку, содержащую п витков, то формула основного закона электромагнитной индукции будет выглядеть так: .
Единица магнитного потока Ф — вебер (Вб): 1В6 = = 1В-с.
Из основного закона следует смысл размерности: 1 вебер — это величина такого магнитного потока, который, уменьшаясь до нуля за одну секунду, через замкнутый контур наводит в нем ЭДС индукции 1 В.
Классической демонстрацией основного закона электромагнитной индукции является первый опыт Фарадея: чем быстрее перемещать магнит через витки катушки, тем больше возникает индукционный ток в ней, а значит, и ЭДС индукции. Зависимость направления индукционного тока от характера изменения магнитного поля через замкнутый контур в 1833 г. опытным путем установил русский ученый Ленц. Он сформулировал правило, носящее его имя. Индукционный ток имеет такое направление, при котором его магнитное поле стремится скомпенсировать изменение внешнего магнитного потока через контур.

Билет № 8.

от класса до кухни — Наука в школе

Автор (ы): Пауло Андре, Ана Рита Бастос и Руте Феррейра

Изучите электромагнитную индукцию и одно из ее широко известных приложений — индукционную плиту — с помощью этих практических занятий.

Многие современные устройства основаны на электромагнитной индукции.Закон индукции Фарадея, сформулированный в 1831 году, описывает, как переменное магнитное поле индуцирует электродвижущую силу (ЭДС). Применения этого закона включают:

  • генераторы, которые производят большую часть электроэнергии, потребляемой во всем мире
  • Поезда на магнитной подушке
  • индукционные плиты на кухне
  • звукосниматели для электрогитары
  • Коврики для беспроводной передачи энергии
  • , используемые для зарядки мобильных устройств.

Преподавание закона индукции Фарадея в средней школе является сложной задачей.Учащиеся в возрасте 16–19 лет должны применять как математические вычисления, так и концептуальное понимание, чтобы исследовать науку, лежащую в основе электромагнитной индукции, и исследовать ее применение в повседневной жизни.

В этом упражнении студенты получают непосредственный опыт работы с электромагнитной индукцией. Эти задания подходят для студентов-физиков в возрасте 16–19 лет, и вместе они занимают около часа, хотя для подготовки к заданию 2 перед уроком требуется дополнительное время.

Индукционная плита
PxHere — CC0

По окончании занятия студенты должны уметь:

  • понять, с экспериментальной точки зрения, закон индукции Фарадея
  • идентифицируют параметры, которые влияют на применение закона индукции Фарадея.
  • исследовать взаимосвязь между изменениями магнитного потока и наведенной электродвижущей силой
  • построить электрический генератор, используя катушку и индукционную плиту для питания светодиода.

Закон индукции Фарадея гласит, что изменение магнитной среды катушки с проволокой вызывает электродвижущую силу (ЭДС), представленную как ε :

, где Н, — количество витков катушки, а Φ — магнитный поток через катушку.

Если масштаб времени небольшой, производная d Φ / d t может быть аппроксимирована Δ Φ / Δ t . Это приближение особенно полезно, если студенты недостаточно знакомы с дифференциальным исчислением.

Магнитный поток зависит от площади катушки A , напряженности магнитного поля B и угла θ , образованного между силовыми линиями магнитного поля и вектором, нормальным (под углом 90 °) к плоскости катушки:

Любое изменение напряженности магнитного поля, площади катушки или угла приводит к наведенной ЭДС, которую можно измерить с помощью гальванометра и использовать для демонстрации принципов индукции.Эта установка показана на рисунке 1.

Рисунок 1: Электрическая цепь с гальванометром и квадратной катушкой в ​​магнитном поле с напряженностью B и направлением.
Изображение любезно предоставлено Пауло Андре

Действие 1: свободно падающий магнит

В этом эксперименте, проводимом учениками, магнитный диполь пропускается через катушку, вызывая ЭДС, вызывая кратковременное изменение интенсивности магнитного поля. Для наблюдения за изменением ЭДС катушку можно подключить к гальванометру или светодиоду (LED).Использование светодиода (рис. 2) имеет то преимущество, что дает видимый выходной сигнал (вспышку света), когда наведенная ЭДС превышает пороговое значение (~ 1,5 В).

Рис. 2. Эксперимент со свободно падающим магнитом с использованием светодиода
Изображение любезно предоставлено Пауло Андре.

Материалы

  • Катушка с N> 10 000 витков
  • Маленький стержневой магнит
  • светодиод
  • Линейка
  • Смартфон (для съемок свободного падения)

Процедура

  1. Подсоедините клеммы катушки к контактным площадкам светодиода.Если требуется пайка, этот шаг должен быть выполнен учителем заранее.
  2. С помощью линейки поместите магнит на 20 см выше центра катушки (и выровняйте с центром катушки).
  3. Настройте камеру смартфона для записи события, желательно в замедленной съемке.
  4. Отпустите магнит и посмотрите на светодиод (рис. 3).
  5. Поэкспериментируйте с освобождением катушки с разной высоты над катушкой. Какие различия, если они есть, вы замечаете?
  6. Теперь поверните катушку на 180º и повторите эксперимент.Наблюдаются ли какие-либо изменения?
Рис. 3. Кадры видеоролика, на котором показан свободно падающий магнит. Последний кадр —
, когда магнит находится внутри катушки, а светодиод светится (красный).

Изображение любезно предоставлено Пауло Андре

Обсуждение

Учителя могут обсудить со студентами следующие вопросы, чтобы изучить ключевые концепции:

  • Как интенсивность излучения светодиода зависит от расстояния, на котором выпущен магнит?
  • При одинаковом расстоянии падения поворот катушки на 180 ° приводит к разной интенсивности излучения светодиода?

Учащиеся могут посмотреть свои видеозаписи эксперимента, чтобы найти доказательства своих ответов, или просмотреть предоставленное видео.В реальной жизни процесс происходит очень быстро, поэтому для прояснения деталей необходимо замедленное видео.

Пояснение

Эксперимент должен показать четкую разницу в интенсивности излучения светодиода по мере увеличения начального расстояния между магнитом и катушкой, что приводит к увеличению скорости магнита и большей скорости изменения магнитного потока.

Светодиод является поляризованным устройством, что означает, что он излучает свет только в том случае, если приложенная ЭДС положительна (когда электрический ток может течь).Положительная или отрицательная ЭДС зависит от направления движения магнита (какой полюс магнита в данный момент движется через катушку) и от того, в каком направлении катушка подключена к контактным площадкам светодиода.

Мы можем увидеть это более подробно в моделировании, показанном на рисунке 4e. Здесь ЭДС положительна только приблизительно в течение половины периода времени, в течение которого магнит находится внутри катушки, и что положительная ЭДС имеет более высокое максимальное значение, чем отрицательная ЭДС, из-за ускорения магнита при прохождении через катушку.

Расширение деятельности 1: математическое моделирование

Хотя мигающий светодиод указывает на наведенную ЭДС в результате изменения магнитного потока, он не дает количественных значений. Мы можем более точно определить изменяющиеся значения магнитного потока при прохождении магнита через катушку, используя следующее уравнение:

, где y представляет координату магнита, y 0 — координату центра катушки, B max — максимальное значение напряженности магнитного поля, а σ — параметр, описывающий спад напряженности магнитного поля.

Здесь мы делаем следующие предположения или приближения:

  • Длина магнита мала по сравнению с катушкой
  • его вектор магнитного поля совмещен с его продольной осью
  • Φ (магнитный поток через катушку) является максимальным, когда геометрические центры катушки и магнита совпадают.

Параметр σ (расстояние для B ~ 37% от B max ) можно принять как находящийся в диапазоне 5–10 мм, полную процедуру экспериментального определения можно найти в работе.[1].

Для магнита в свободном падении смещение y магнита в момент времени t связано с ускорением свободного падения g посредством уравнения 4:

В таблице 1 приведен пример используемых параметров.

Таблица 1: Примеры значений параметров, использованных для получения результатов на Рисунке 4.
Параметры Символ Значение Блок
Площадь змеевика А 250 мм 2
Число витков катушки N 12000
Максимальная напряженность магнитного поля B макс 3 мТ
Распад магнитного поля σ 10 мм
Координата центра катушки y 0 200 мм

Используя значения в таблице 1, уравнениях 3 и 4 и прилагаемой электронной таблице, мы можем создать математическую модель, которая графически отображает эти изменения, как показано в результатах и ​​графиках на рисунке 4. [2] Здесь магнитный поток через катушку и наведенная ЭДС показаны как функция положения и времени. Магнит отпускается при y = 0 и t = 0, и он падает в положительном направлении оси. Катушка центрируется в позиции y 0 .

Рис. 4: а) Схема эксперимента. b – e) Смоделированные значения для магнита в свободном падении: магнитный поток как функция положения (b) и времени (d) и ЭДС как функция положения (c) и времени (e)
Изображение любезно предоставлено Пауло Андре

Обсуждение

Используя рисунок 4 и таблицу, учителя могут обсудить со студентами следующие вопросы:

  • Как изменяется магнитный поток по мере приближения магнита к центру катушки? (Он увеличивается, а после прохождения центра следует уменьшение — см. Рис.4б.)
  • На рисунке 4c, что происходит с ЭДС, когда магнит проходит через центр катушки? (Знак меняется с отрицательного на положительный.)
  • На рисунке 4e форма кривой ЭДС симметрична до и после прохождения магнита через центр катушки? (Нет, он асимметричный, потому что магнит находится в свободном падении, поэтому его скорость увеличивается со временем, а скорость изменения магнитного потока и наведенной ЭДС также увеличивается со временем.)

Мероприятие 2: индукционная плита

Реальное применение закона Фарадея — приготовление пищи с использованием индукционной плиты, где сковорода нагревается за счет электрической индукции, а не за счет теплопроводности от пламени или электрической плиты.Индукционные плиты генерируют тепло внутри самой посуды, что делает этот метод приготовления более эффективным. Однако все сковороды должны быть изготовлены из ферромагнитного металла (обычно из чугуна или нержавеющей стали).

Индукционная плита имеет катушку, питаемую переменным электрическим током, под керамической пластиной. Переменный ток создает колеблющееся магнитное поле, которое индуцирует колеблющийся магнитный поток в основании кастрюли, установленной на плиту. Это создает электрический ток (называемый вихревым током) в дне посуды, нагревая его.

В индукционных плитах напряженность магнитного поля обычно мала (~ 100 мТл), но колеблется с высокой частотой (27 кГц). Это означает, что скорость изменения напряженности магнитного поля очень высока, что приводит к высоким значениям наведенной ЭДС и, следовательно, для произведенного нагрева.

В этом упражнении студенты исследуют наведенные ЭДС вокруг индукционной плиты, снова используя светодиод, подключенный к катушке.

Материалы

  • Кухонная индукционная плита
  • Светодиоды (разные цвета свечения)
  • 0.Медный провод диаметром 2 мм для изготовления катушки
  • Карандаш
  • Бумага
  • Клейкая лента

Указание по безопасности

Хотя индукционная плита не нагревается, сковорода и вода нагреваются, поэтому учеников следует предупредить, чтобы они не прикасались к ней, и следует позаботиться о том, чтобы ручка сковороды не мешала и ее нельзя было легко ударить при переноске. вне эксперимента. Следует проявлять особую осторожность, если ученики должны выполнять этап пайки самостоятельно, и это должно выполняться только под пристальным наблюдением учителя.

Процедура

  1. Возьмите карандаш, накройте его листом бумаги (это будет внутренняя часть катушки) и намотайте на него 300–400 витков проволоки.
  2. Наклейте ленту, чтобы закрыть провод, и удерживайте ее на месте; затем удалите карандаш.
  3. Удалите эмаль с медных проводов на каждом конце катушки.
  4. Припаяйте два светодиода к медным проводам встречно-параллельно (параллельно, но с обратной полярностью относительно друг друга). Учитель может выполнить этот шаг или ученики могут сделать это под присмотром учителя.
  5. Поставьте кастрюлю с водой в центр индукционной плиты.
  6. Поместите змеевик рядом с поддоном.
  7. Включите варочную панель, начиная с низкой интенсивности (рис. 5).
  8. Переместите змеевик по окружающим участкам варочной панели. Попробуйте выяснить, какие изменения приводят к тому, что светодиод становится ярче или тусклее.
Рис. 5. Змеевик рядом с индукционной плитой, с выключенной (слева) и включенной (справа) варочной панелью
Изображение любезно предоставлено Пауло Андре

Обсуждение

Учителя могут обсудить со студентами следующие вопросы, чтобы изучить ключевые концепции.

  • Как яркость светодиода зависит от расстояния до варочной панели?
  • Поворот катушки в том же положении приведет к изменению яркости светодиода?

Ученики должны обнаружить, что магнитное поле в основном ограничено областью кастрюли, а интенсивность поля быстро уменьшается по мере удаления от варочной панели. Вращение катушки приводит к изменению ЭДС в соответствии с уравнением (2) из-за изменения угла между катушкой и силовой линией магнитного поля.

Майкл Фарадей (1791–1867)

Портрет Майкла Фарадея. Автор
Томас Филлипс (1842)

Общественное достояние

Майкл Фарадей был британским ученым, который изложил принципы, лежащие в основе электромагнитной индукции. Хотя Фарадей не получил формального образования, он стал одним из величайших ученых-первооткрывателей в истории. Единица электрической емкости, фарад (F), названа в его честь, и во многом благодаря усилиям Фарадея электричество стало практичным для широкого использования.Некоторые концепции, которые он извлек из экспериментов, например, силовые линии магнитного поля, стали важными теоретическими идеями в физике, положив начало современной теории электромагнетизма.

Фарадей работал в лаборатории Королевского института в Лондоне. В 1831 году он продемонстрировал принцип индукции: это позволило разработать динамо-машину (или генератор), вырабатывающую электричество механическими средствами. В 1845 году Фарадей также установил, что сильное магнитное поле может вращать плоскость поляризации света (теперь известное как эффект Фарадея), показывая лежащую в основе взаимосвязь между магнетизмом и светом.

Фарадей прекратил исследовательскую работу в 1855 году, но продолжал работать лектором до 1861 года.

Благодарности

Эта работа была разработана в рамках проекта CICECO-Aveiro Institute of Materials (UIDB / 50011/2020 & UIDP / 50011/2020), Instituto de Telecomunicações (UIDB / 50008/2020-UIDP / 50008/2020) и WinLEDs (POCI-01-0145-FEDER-030351) финансируется из национальных фондов через FCT / MEC и, при необходимости, софинансируется FEDER в рамках Партнерства PT2020 через Европейский фонд регионального развития (ERDF) в рамках Программы повышения операционной конкурентоспособности и интернационализации ( POCI).

Скачать

Загрузить эту статью в формате PDF

Ссылки

[1] Enrique A et al. (2015) Измерение магнитного поля малых магнитов с помощью смартфона: очень экономичная лабораторная практика для вводных курсов физики. Европейский журнал физики 36 : 1–11. DOI: 10.1088 / 0143-0807 / 36/6/065002

[2] Амрани Д. (2005) Электродвижущая сила: закон индукции Фарадея рассматривается как свободно падающий магнит. Физическое образование 40 : 313–314. DOI: 10.1088 / 0031-9120 / 40/4 / F02

Ресурсы

Автор (ы)

Пауло Андре является профессором телекоммуникационной инженерии в Высшем техническом институте Лиссабонского университета, Португалия. Он имеет докторскую степень в области инженерной физики, и его исследовательские интересы включают устройства и системы фотоники.

Ана Бастос — младший научный сотрудник Департамента физики и Института материалов CICECO Университета Авейру, Португалия.В ее специальные исследовательские интересы входят системы оптоэлектроники, интегрированная оптика и оптическая связь.

Руте Феррейра — доцент кафедры физики Университета Авейру, Португалия. Она координирует исследования в области информационных и коммуникационных технологий в Институте материалов CICECO Университета Авейру. В настоящее время ее научные интересы сосредоточены на органических / неорганических гибридах с перспективными приложениями в областях оптоэлектроники / зеленой фотоники (твердотельное освещение и интегрированная оптика) и фотовольтаики (люминесцентные солнечные концентраторы).

Лицензия

Что такое закон Фарадея? Законы электромагнитной индукции

Закон электромагнитной индукции Фарадея

Электромагнетизм

Взаимодействие между магнитным полем и электрическим током называется электромагнетизмом. Проводники с током создают магнитное поле, когда через них проходит ток. Движение электронов в проводнике приведет к возникновению электрического тока (дрейфующих электронов), который возникает в результате ЭДС, возникающей в проводнике.

ЭДС, возникающая в проводнике, может быть в форме энергии, хранящейся в химической энергии или магнитном поле. Проводники с током, помещенные в магнитное поле, будут испытывать механическую силу, в то время как проводники, помещенные в магнитное поле, будут перемещать электроны, что приведет к возникновению электрического тока.

Field Flux

Два магнита с разными полюсами будут притягиваться друг к другу, в то время как магниты с одинаковыми полюсами будут отталкивать друг друга (то же самое происходит с электрическими зарядами).Каждый магнит окружен силовым полем и представлен воображаемыми линиями, исходящими от северного полюса магнита, переходящего в южный полюс того же магнита.

Прочтите важные термины, связанные с потоком поля и магнитом. Заполните формулы здесь

«Линии, соединяющие северный и южный полюсы магнита, которые представляют силовое поле, которое связывает катушки в трансформаторе, называются магнитным потоком».

Электромагнитная индукция

Электромагнитная индукция — это явление, которое объясняет, как ЭДС и ток индуцируются или могут индуцироваться в катушке при взаимодействии катушки и магнитного поля.Это явление «электромагнитной индукции» объясняется законами электромагнитной индукции Фарадея. Направление наведенной ЭДС в катушке или индукторах объясняется законом Ленца и правилом правой руки Флеминга.

Похожие сообщения:

Законы электромагнитной индукции Фарадея

После того, как Андре Мари Ампер (французский математик и физик, известный как отец электромагнетизма) и другие исследовали магнитный эффект тока, Майкл Фарадей попробовал обратное.В ходе своей работы в 1831 году он открыл принцип электромагнитной индукции, согласно которому при изменении магнитного поля, в которое была помещена катушка или индуктор, в катушке индуцировалась ЭДС.

Это происходило только всякий раз, когда он перемещал катушку или магнит, которые он использовал в эксперименте. ЭДС индуцировалась в катушке только тогда, когда происходило изменение потока поля (если катушка зафиксирована, перемещение магнита по направлению к катушке или от нее вызывает индукцию ЭДС). Таким образом, законы электромагнитной индукции Фарадея гласят:

Первый закон Фарадея

Первый закон электромагнитной индукции Фарадея гласит, что «ЭДС индуцируется в катушке, когда происходит изменение потока, связанного с катушкой».

Другими словами, всякий раз, когда поток, связанный или связанный с цепями, изменяется. в цепи индуцируется E.M.F. Эта ЭДС длится только до тех пор, пока происходят изменения. Индуцированная ЭДС изменяется как скорость изменения потока.

Второй закон Фарадея

Второй закон электромагнитной индукции Фарадея гласит, что «величина наведенной ЭДС в катушке прямо пропорциональна скорости изменения магнитного потока, связанного с катушкой».

Другими словами, E.МФ, индуцированная в электрической цепи, пропорциональна скорости изменения во времени потока магнитной индукции, связанного с цепью. Величина наведенного ЭДС прямо пропорциональна скорости изменения тока. Короче говоря, чем больше потокосцепление к катушке или проводнику, тем больше наведенная ЭДС (dΦ / dt).

Законы электромагнитной индукции Фарадея можно математически записать в виде уравнения следующим образом.

e = N d Φ / d t

Где

  • e = индуцированная ЭДС
  • N = количество витков
  • dΦ = изменение потока
  • dt = изменение во времени

Формула и уравнение закона электромагнитной индукции Фарадея:

Предположим, что катушка содержит N витков, а магнитный поток изменяется от начального значения «Φ 1 » до конечного значения «Φ ». 2 »за время« t »секунд.Имейте в виду, что потокосцепление — это произведение связанного потока на количество витков в катушке. т.е.

Начальные потокосцепления = NΦ 1

Конечные потокосцепления = NΦ 2

Индуцированная ЭДС

e = NΦ 1 — NΦ 2 / t… wb / s или напряжение

Уравнение ЭДС «e» преобразовано в дифференциальную форму

e = d / dt (NΦ)… вольт

e = N (dΦ / dt)… вольт

Здесь скорость изменения потока (dΦ) происходит меньше возможное время (dΦ).Знак минус «-» в правой части уравнения показывает, что индуцированная ЭДС направляет ток в таком направлении, где он противодействует его магнитному эффекту, который произвел ЭДС. Проще говоря, индуцированная ЭДС противодействует причине (изменение тока или движение), которая ее производит (ЭДС). Это явление также известно как закон Ленца.

e = — N (dΦ / dt)… вольт

Наконец, эта формула показывает, что ЭДС, индуцированная в катушке, равна скорости изменения потока (dΦ / dt), умноженной на количество витков (Н) в этой катушке.то есть

e = N (dΦ / dt)… вольт

Объяснение и действие закона Фарадея

На следующем рисунке показаны различные сценарии работы закона Фарадея.

Рис. 1.A показывает, что когда магнит движется вправо, магнитное поле изменяется по отношению к катушке, и индуцируется ЭДС.

На рис. 1.B показано, что чем быстрее магнит движется вправо, тем быстрее изменяется магнитное поле по отношению к катушке, и индуцируется большая ЭДС.

На рис. 2.A показано, что магнит движется через катушку и индуцирует ЭДС.

Рис. 2.B показывает, что магнит движется с той же скоростью через катушку с большим количеством витков (петель) и индуцирует большую ЭДС.

На рис. 2 показана основная демонстрация второго закона Фарадея, т.е. величина наведенной ЭДС прямо пропорциональна количеству мелодий в катушке.

Применение закона Фарадея

Самый мощный закон электромагнитной индукции Майкла Фарадея используется в различных приложениях, таких как электрические машины, медицинские поля, промышленность и т. Д.Некоторые из них следующие.

  • Электрические трансформаторы (силовые и распределительные т / ф), асинхронные двигатели, генераторы и генераторы переменного тока (для выработки электроэнергии) работают по принципу взаимной индукции, то есть по закону Фарадея.
  • Принцип действия электромагнитного расходомера и индукционной плиты основан на законе электромагнетизма Фарадея.
  • Он также используется в уравнении Максвелла, основанном на силовых линиях.
  • Закон Фарадея также применим к развлечениям и музыкальным инструментам e.грамм. электрическое пианино, скрипка, электрогитара и т. д.
  • Магнитная индукция на основе закона Фарадея, используемая в электрических и гибридных транспортных средствах, и транскраниальная магнитная стимуляция.
  • Компьютер HD (жесткие диски) и графические планшеты работают на магнитной индукции, которая основана на законе Фарадея.

Решенный пример закона электромагнетизма Фарадея

Пример:

Примените закон Фарадея, чтобы найти наведенное напряжение или ЭДС на катушке со 100 витками, которая находится в магнитном поле и изменяется со скоростью 5 Вт / с.

Решение:

Приведенные данные:

  • Число витков = 100
  • Скорость изменения магнитного поля = 5 Вт / с.

Подставляем значения в уравнение закона Фарадея

e = N (dΦ / dt)

e = 100 x (5)

e = 500V.

Похожие сообщения

Закон электромагнитной индукции Фарадея

Название Закон Фарадея Электромагнитной индукции дано в честь известного ученого Майкла Фарадея в 1930-х годах.Он показывает взаимосвязь между электрическим напряжением и изменяющимся магнитным полем. Закон Фарадея об электромагнитной индукции гласит, что «величина напряжения прямо пропорциональна скорости изменения магнитного потока». это означает, что напряжение индуцируется в цепи, когда есть относительное движение между магнитным полем и проводником.

Электромагнитная индукция

В замкнутой цепи, когда протекает ток и индуцируется ЭДС, поэтому явление, при котором ЭДС индуцируется в цепи при изменении магнитного потока, связанного с ней, называется Электромагнитная индукция.

Это можно пояснить на примере

Рассмотрим катушку с большим количеством витков, к которой подключен гальванометр

Случай 1: — Катушка неподвижна, а магнит движется

Когда постоянный стержневой магнит приближается к катушке (положение 2) или от катушки (положение 1), как показано на рисунке выше, в гальванометре происходит отклонение. В обоих случаях прогибы противоположны.

Случай 2: — Катушка движется, а магнит неподвижен.

Если стержневой магнит остается неподвижным, а катушка приближается к магниту (положение 1) или от магнита (положение 2), в гальванометре произойдет отклонение.

В обоих случаях направление иглы будет противоположным. Это можно объяснить тем, что предположим, что если поднести магнит ближе, чем игла будет отклоняться вправо, а если отодвинуть от магнита, она покажет отклонение с левой стороны.

Случай 3: — Когда магнит и катушка неподвижны

Когда и магнит, и катушка остаются неподвижными, в катушке не будет отклонений, независимо от того, какой поток связан с катушкой

Проанализированы следующие точки

  1. Отклонение стрелки гальванометра указывает на то, что в катушке наведена ЭДС. Отклонение происходит только при изменении магнитного потока, связанного с цепью. то есть либо магнит, либо катушка находятся в движении.
  2. Направление наведенной ЭДС в катушке зависит от направления магнитного поля и направления движения катушки.

Закон Ленца — Физический колледж

Цели обучения

  • Рассчитайте ЭДС, ток и магнитные поля, используя закон Фарадея.
  • Объясните физические результаты Закона Ленца.

Закон Фарадея и Ленца

Эксперименты Фарадея показали, что ЭДС , вызванная изменением магнитного потока, зависит только от нескольких факторов.Во-первых, ЭДС прямо пропорциональна изменению магнитного потока. Во-вторых, ЭДС является наибольшей, когда изменение во времени наименьшее, то есть ЭДС обратно пропорциональна. Наконец, если в катушке есть витки, будет создаваться ЭДС, которая в разы больше, чем для одиночной катушки, так что ЭДС прямо пропорциональна. Уравнение для ЭДС, вызванной изменением магнитного потока:

Это соотношение известно как закон индукции Фарадея.Обычно единицами измерения ЭДС являются вольты.

Знак минус в законе индукции Фарадея очень важен. Минус означает, что ЭДС создает ток I и магнитное поле B, которые препятствуют изменению магнитного потока — это известно как закон Ленца . Направление (обозначенное знаком минус) ЭДС настолько важно, что оно названо законом Ленца в честь русского Генриха Ленца (1804–1865), который, подобно Фарадею и Генри, независимо исследовал аспекты индукции. Фарадей знал о направлении, но Ленц так ясно изложил его, что ему приписывают его открытие.(См. (Рисунок).)

(а) Когда стержневой магнит вставляется в катушку, сила магнитного поля в катушке увеличивается. Ток, наведенный в катушке, создает другое поле в направлении, противоположном направлению стержневого магнита, чтобы противодействовать увеличению. Это один из аспектов закона Ленца — индукция препятствует любому изменению потока . (b) и (c) — две другие ситуации. Убедитесь сами, что показанное направление индуцированного тока действительно противостоит изменению магнитного потока и что показанное направление тока соответствует RHR-2.

Стратегия решения проблем закона Ленца

Чтобы использовать закон Ленца для определения направлений индуцированных магнитных полей, токов и ЭДС:

  1. Сделайте набросок ситуации для использования при визуализации и записи направлений.
  2. Определить направление магнитного поля B.
  3. Определите, увеличивается или уменьшается поток.
  4. Теперь определите направление индуцированного магнитного поля B. Оно противодействует изменению магнитного потока путем добавления или вычитания из исходного поля.
  5. Используйте RHR-2, чтобы определить направление индуцированного тока I, ответственного за индуцированное магнитное поле B.
  6. Направление (или полярность) наведенной ЭДС теперь будет управлять током в этом направлении и может быть представлено как ток, выходящий из положительного вывода ЭДС и возвращающийся к его отрицательному выводу.

Для практики примените эти шаги к ситуациям, показанным на (Рисунок), и другим, которые являются частью следующего текстового материала.

Применение электромагнитной индукции

Существует множество применений закона индукции Фарадея, которые мы исследуем в этой и других главах. На этом этапе позвольте нам упомянуть несколько, которые связаны с хранением данных и магнитными полями. Очень важное приложение связано с аудио и видео , записывающими кассеты . Пластиковая лента, покрытая оксидом железа, проходит мимо записывающей головки. Эта записывающая головка представляет собой круглое железное кольцо, на которое намотана катушка с проволокой — электромагнит ((Рисунок)).Сигнал в виде переменного входного тока от микрофона или камеры поступает на записывающую головку. Эти сигналы (которые являются функцией амплитуды и частоты сигнала) создают переменные магнитные поля на записывающей головке. Когда лента движется мимо записывающей головки, ориентация магнитного поля молекул оксида железа на ленте изменяется, таким образом записывая сигнал. В режиме воспроизведения намагниченная лента проходит мимо другой головки, аналогичной по конструкции записывающей головке. Различная ориентация магнитного поля молекул оксида железа на ленте индуцирует ЭДС в проволочной катушке в воспроизводящей головке.Затем этот сигнал отправляется на громкоговоритель или видеоплеер.

Головки для записи и воспроизведения, используемые с аудио- и видеомагнитными лентами. (кредит: Стив Джурветсон)

Аналогичные принципы применимы и к жестким дискам компьютеров, но с гораздо большей скоростью. Здесь записи находятся на вращающемся диске с покрытием. Исторически считывающие головки создавались по принципу индукции. Однако входная информация передается в цифровой, а не аналоговой форме — на вращающемся жестком диске записывается серия нулей или единиц.Сегодня большинство считывающих устройств с жестких дисков не работают по принципу индукции, а используют технологию, известную как гигантское магнитосопротивление . (Открытие того факта, что слабые изменения магнитного поля в тонкой пленке из железа и хрома могут приводить к гораздо большим изменениям электрического сопротивления, было одним из первых крупных успехов нанотехнологии.) Еще одно применение индукции можно найти на магнитной полосе на магнитной полосе. на оборотной стороне вашей личной кредитной карты, которая использовалась в продуктовом магазине или в банкомате.Это работает по тому же принципу, что и аудио- или видеопленка, упомянутая в последнем абзаце, в которой голова считывает личную информацию с вашей карты.

Другое применение электромагнитной индукции — это когда электрические сигналы должны передаваться через барьер. Рассмотрим кохлеарный имплант , показанный ниже. Звук улавливается микрофоном на внешней стороне черепа и используется для создания переменного магнитного поля. Ток индуцируется в приемнике, закрепленном в кости под кожей, и передается на электроды во внутреннем ухе.Электромагнитная индукция может использоваться и в других случаях, когда электрические сигналы должны передаваться через различные среды.

Электромагнитная индукция, используемая при передаче электрического тока через среды. Устройство на голове ребенка индуцирует электрический ток в приемнике, закрепленном в кости под кожей. (кредит: Бьорн Кнетч)

Еще одна современная область исследований, в которой электромагнитная индукция успешно реализуется (и имеет значительный потенциал), — это транскраниальное магнитное моделирование.Множество расстройств, включая депрессию и галлюцинации, можно объяснить нерегулярной локальной электрической активностью в головном мозге. В транскраниальной магнитной стимуляции быстро меняющееся и очень локализованное магнитное поле помещается рядом с определенными участками, идентифицированными в головном мозге. В идентифицированных участках индуцируются слабые электрические токи, которые могут привести к восстановлению электрических функций в тканях мозга.

Апноэ сна («остановка дыхания») поражает как взрослых, так и младенцев (особенно недоношенных детей, и это может быть причиной внезапной детской смерти [SID]).У таких людей дыхание может неоднократно останавливаться во время сна. Прекращение действия более чем на 20 секунд может быть очень опасным. Инсульт, сердечная недостаточность и усталость — вот лишь некоторые из возможных последствий для человека, страдающего апноэ во сне. У младенцев проблема заключается в задержке дыхания на это более длительное время. В одном из типов мониторов, предупреждающих родителей о том, что ребенок не дышит, используется электромагнитная индукция. В проводе, обмотанном вокруг груди младенца, проходит переменный ток. Расширение и сжатие грудной клетки младенца во время дыхания изменяет площадь спирали.В расположенной рядом катушке датчика индуцируется переменный ток из-за изменения магнитного поля исходного провода. Если ребенок перестанет дышать, наведенный ток изменится, и родители могут быть предупреждены.

Установление соединений: сохранение энергии

Закон Ленца — это проявление сохранения энергии. Индуцированная ЭДС создает ток, который противодействует изменению потока, потому что изменение потока означает изменение энергии. Энергия может входить или уходить, но не мгновенно.Закон Ленца — следствие. Когда изменение начинается, закон гласит, что индукция противодействует и, таким образом, замедляет изменение. Фактически, если бы индуцированная ЭДС была в том же направлении, что и изменение потока, была бы положительная обратная связь, которая не давала бы нам бесплатную энергию без видимого источника — закон сохранения энергии был бы нарушен.

Расчет ЭДС: насколько велика индуцированная ЭДС?

Рассчитайте величину наведенной ЭДС, когда магнит на (Рисунок) (a) вдавливается в катушку, учитывая следующую информацию: одноконтурная катушка имеет радиус 6.00 см, а среднее значение (это дано, поскольку поле стержневого магнита сложное) увеличивается с 0,0500 Тл до 0,250 Тл за 0,100 с.

Стратегия

Чтобы найти величину ЭДС , мы используем закон индукции Фарадея, как указано, но без знака минус, указывающего направление:

Решение

Нам дано это и, но мы должны определить изменение потока, прежде чем сможем найти ЭДС. Поскольку площадь петли фиксирована, мы видим, что

Теперь, поскольку было дано, меняется с 0.От 0500 до 0,250 Тл. Площадь петли составляет. Таким образом,

Ввод определенных значений в выражение для ЭДС дает

Обсуждение

Хотя это напряжение легко измерить, его явно недостаточно для большинства практических приложений. Больше петель в катушке, более сильный магнит и более быстрое движение делают индукцию практическим источником напряжения.

Исследования PhET: Электромагнитная лаборатория Фарадея

Поиграйте с стержневым магнитом и катушками, чтобы узнать о законе Фарадея.Поднесите стержневой магнит к одной или двум катушкам, чтобы лампочка загорелась. Просмотрите силовые линии магнитного поля. Измеритель показывает направление и величину тока. Просмотрите силовые линии магнитного поля или используйте измеритель, чтобы показать направление и величину тока. Вы также можете играть с электромагнитами, генераторами и трансформаторами!

Сводка раздела

  • Закон индукции Фарадея гласит, что ЭДС , вызванная изменением магнитного потока, равна

, когда поток меняется во времени.

Если в катушке индуцируется ЭДС, это ее количество витков. Знак минус означает, что ЭДС создает ток и магнитное поле, которые противодействуют изменению магнитного потока. — это противоречие известно как закон Ленца. работает с большими магнитами, иногда помещает голову в сильное поле. Она сообщает, что у нее кружится голова, когда она быстро поворачивает голову. Как это может быть связано с индукцией? Ускоритель частиц отправляет заряженные частицы с высокой скоростью по откачанной трубе.Объясните, как катушка с проволокой, намотанная вокруг трубы, может обнаруживать прохождение отдельных частиц. Нарисуйте график выходного напряжения катушки при прохождении через нее одиночной частицы. Задачи и упражнения Ссылаясь на (Рисунок) (а), каково направление тока, индуцируемого в катушке 2: (а) Если ток в катушке 1 увеличивается? (b) Если ток в катушке 1 уменьшается? (c) Если ток в катушке 1 постоянный? Ясно покажите, как вы следуете шагам, изложенным в Стратегии решения проблем по закону Ленца.(а) Катушки лежат в одной плоскости. (b) Провод находится в плоскости катушки (a) CCW (b) CW (c) Нет индуцированного тока Ссылаясь на (рисунок) (b), каково направление тока, наведенного в катушке: (a) Если ток в проводе увеличивается? (б) Если ток в проводе уменьшится? (c) Если ток в проводе внезапно меняет направление? Ясно покажите, как вы следуете этапам стратегии решения проблем для закона Ленца. Ссылаясь на (рисунок), каковы направления токов в катушках 1, 2 и 3 (предположим, что катушки лежат в плоскости цепи): (а) Когда переключатель в первый раз замыкается? (б) Когда переключатель был замкнут в течение длительного времени? (c) Сразу после размыкания переключателя? (a) 1 CCW, 2 CCW, 3 CW (b) 1, 2 и 3 отсутствие тока (c) 1 CW, 2 CW, 3 CCW Повторите предыдущую проблему с аккумулятором в обратном порядке.Убедитесь, что единицы измерения / — вольт. То есть, покажите, что. Предположим, катушка с 50 витками лежит в плоскости страницы в однородном магнитном поле, которое направлено внутрь страницы. Катушка изначально имела площадь. Он растягивается, чтобы не было площади за 0,100 с. Каково направление и величина наведенной ЭДС, если однородное магнитное поле имеет напряженность 1,50 Тл? (А) Техник МРТ перемещает руку из области очень низкой напряженности магнитного поля в поле 2,00 Тл сканера МРТ пальцами. указывая в направлении поля.Найдите среднюю ЭДС, индуцированную в его обручальном кольце, учитывая его диаметр 2,20 см и предполагая, что для его перемещения в поле требуется 0,250 с. (b) Обсудите, изменит ли этот ток значительно температуру кольца. (a) 3,04 мВ
(b) В качестве нижнего предела для кольца оцените R = 1,00 мОм. Передаваемое тепло составит 2,31 мДж. Это незначительное количество тепла. Комплексные концепции Ссылаясь на ситуацию в предыдущей задаче: (а) Какой ток индуцируется в кольце, если его сопротивление равно 0.0100? (б) Какая средняя мощность рассеивается? (c) Какое магнитное поле индуцируется в центре кольца? (d) Каково направление индуцированного магнитного поля относительно поля МРТ? ЭДС индуцируется вращением катушки с 1000 витками диаметром 20,0 см в магнитном поле Земли. Какая средняя ЭДС индуцируется, учитывая, что плоскость катушки изначально перпендикулярна полю Земли и повернута параллельно полю за 10,0 мс? 0,157 ВА Радиус 0,250 м, 500-витковая катушка вращается на одну четверть оборота в 4.17 мс, изначально имеющая плоскость, перпендикулярную однородному магнитному полю. (Это 60 об / с.) Найдите напряженность магнитного поля, необходимую для индукции средней ЭДС в 10 000 В. Приблизительно как ЭДС, индуцированная в контуре (Рисунок) (b), зависит от расстояния до центра петля из провода? пропорциональна Интегрированным концепциям (а) Молния создает быстро меняющееся магнитное поле. Если болт ударяется о землю вертикально и действует как ток в длинном прямом проводе, он вызывает напряжение в петле, выровненной, как показано на (Рисунок) (b).Какое напряжение индуцируется в петле диаметром 1,00 м в 50,0 м от удара молнии, если ток падает до нуля? (б) Обсудите обстоятельства, при которых такое напряжение могло бы привести к заметным последствиям. Глоссарий Закон индукции Фарадея средство вычисления ЭДС в катушке, обусловленной изменением магнитного потока, которое, согласно закону Ленца, является знаком минус в законе Фарадея, означающим, что ЭДС индуцирована в катушка противодействует изменению магнитного потока

Электромагнитная индукция | HowStuffWorks

В магнетизме есть что-то почти волшебное.В детстве мы были очарованы способностью магнита воздействовать на такие металлы, как железо, никель и кобальт, не касаясь их. Мы узнаем о притяжении и отталкивании между магнитными полюсами и становимся свидетелями формы магнитного поля, сформированного в железных опилках, окружающих стержневой магнит. Физики говорят нам, что электромагнетизм, сила, управляющая электричеством и магнетизмом, во много раз сильнее гравитации. Подвешивание поезда на магнитной подвеске над его путями — яркий пример этой силы.

Как следует из названия «электромагнетизм», электричество и магнетизм очень тесно связаны.Эта взаимосвязь позволяет им влиять друг на друга бесконтактно, как в примере с поездом на магнитной подвеске, или посредством электромагнитной индукции. Электромагнитная индукция возникает, когда цепь с протекающим через нее переменным током генерирует ток в другой цепи, просто будучи размещенной поблизости. Переменный ток — это электричество, протекающее по линиям электропередач и домашней электропроводке, в отличие от постоянного тока, который мы получаем от батарей.

Как одна цепь вызывает ток в другой, не касаясь ее, и какое отношение все это имеет к магнетизму? Прежде чем мы перейдем к этому, нам нужно рассмотреть несколько принципов, связывающих магнетизм и электричество:

  1. Каждый электрический ток имеет окружающее магнитное поле.
  2. У переменного тока есть переменные магнитные поля.
  3. Колеблющиеся магнитные поля заставляют токи течь в проводниках, помещенных в них, что также известно как закон Фарадея.

Сложение этих трех свойств вместе означает, что изменяющийся электрический ток окружен соответствующим изменяющимся магнитным полем, которое, в свою очередь, генерирует изменяющийся электрический ток в проводнике, помещенном внутри него, который имеет собственное магнитное поле… и так далее. Это электромагнитный эквивалент матрешки-матрешки.Таким образом, в случае электромагнитной индукции размещение проводника в магнитном поле, окружающем первый ток, генерирует второй ток.

Индукция — это принцип, который делает возможными электродвигатели, генераторы и трансформаторы, а также предметы, расположенные ближе к дому, такие как перезаряжаемые электрические зубные щетки и устройства беспроводной связи. Если у вас есть рисоварка, скорее всего, вы уже готовите на индукции. Теперь давайте посмотрим, как наведенный ток используется для нагрева индукционных варочных панелей.

Теоретический вывод второго закона электромагнитной индукции Фарадея

Pub. Дата: 28 сентября 2020 г.

Цитируйте эту статью:
Wei Feng. Теоретический вывод второго закона электромагнитной индукции Фарадея. Международный физический журнал . 2020; 8 (4): 120-123. doi: 10.12691 / ijp-8-4-1

[1] Роберт Кингман, С. Кларк Роуленд, Сабин Попеску, Экспериментальное наблюдение закона индукции Фарадея, Am.J. Phys. 2002 (70) 595-560.
[2] А. Сингх, Ю. Н. Мохапатра и С. Кумар, Электромагнитная индукция и демпфирование: количественные эксперименты с использованием интерфейса ПК, Am. J. Phys (2002). 70, 424-427.
[3] Р. Кингман, С. К. Роуленд и С. Попеску, Экспериментальное наблюдение закона индукции Фарадея, Am. J. Phys. 2002 (70), 595-598.
[4] С.К. Джонс, аппарат закона Фарадея для лаборатории первокурсников, Am. J. Phys. (1987) 55, 1148-1150.
[5] Дж. Барберо, Дж. А. Мансанарес и С. Мафе, Индуцированная ЭДС в соленоиде: простая количественная проверка закона Фарадея, Phys. Educ. (1994) 29, 102-105.
[6] К. С. Маклатчи, П. Бакман и Л. Боган, Количественный эксперимент по магнитному разрушению, Am. J. Phys.1993 (61), 1096-1101.
[7] К. Д. Хан, Э. М. Джонсон, А. Броккен и С. Болдуин, Вихретоковое затухание магнита, движущегося по трубе, Am. J. Phys. 1998 (66), 1066-1076.
[8] Р. К. Никлин, Закон Фарадея и качественные эксперименты, Am. J. Phys. (1986) 54, 422-428;
[9] К. С. Маклатчи, П.Бакман, Л. Боган, Количественный эксперимент по магнитному разрушению, Am. J. Phys. (1993) 61, 1096-1101.
[10] J. A. Manzanares, J. Bisquert, G. Garcia-Belmonte и M. Feranandez Alonso, Эксперимент по импульсам магнитной индукции, Am. J. Phys (1994). 62,702-706.
[11] П. Карпена, Измерения скорости посредством магнитной индукции, Ам. J. Phys (1997). 65, 135-140.
[12] Игал Галили, Дов Каплан и Ярон Лехави, Изучение закона электромагнитной индукции Фарадея во вводном курсе физики, Am. J. Phys. (2006) 74, 337.

Закон Ленца — Физика колледжа: OpenStax

Существует множество приложений закона индукции Фарадея, которые мы исследуем в этой и других главах. На этом этапе позвольте нам упомянуть несколько, которые связаны с хранением данных и магнитными полями.Очень важное приложение связано с аудио и видео , записывающими лентами . Пластиковая лента, покрытая оксидом железа, проходит мимо записывающей головки. Эта записывающая головка представляет собой круглое железное кольцо, вокруг которого намотана катушка с проволокой — электромагнит (рис. 2). Сигнал в виде переменного входного тока от микрофона или камеры поступает на записывающую головку. Эти сигналы (которые являются функцией амплитуды и частоты сигнала) создают переменные магнитные поля на записывающей головке.Когда лента движется мимо записывающей головки, ориентация магнитного поля молекул оксида железа на ленте изменяется, таким образом записывая сигнал. В режиме воспроизведения намагниченная лента проходит мимо другой головки, аналогичной по конструкции записывающей головке. Различная ориентация магнитного поля молекул оксида железа на ленте индуцирует ЭДС в проволочной катушке в воспроизводящей головке. Затем этот сигнал отправляется на громкоговоритель или видеоплеер.

Аналогичные принципы применимы и к жестким дискам компьютера, только с гораздо большей скоростью.Здесь записи находятся на вращающемся диске с покрытием. Исторически считывающие головки создавались по принципу индукции. Однако входная информация передается в цифровой, а не аналоговой форме — на вращающемся жестком диске записывается серия нулей или единиц. Сегодня большинство считывающих устройств с жестких дисков не работают по принципу индукции, а используют технологию, известную как гигантское магнитосопротивление . (Открытие того, что слабые изменения магнитного поля в тонкой пленке из железа и хрома могут вызывать гораздо большие изменения электрического сопротивления, было одним из первых крупных успехов нанотехнологии.Еще одно применение индукции можно найти на магнитной полосе на обратной стороне вашей личной кредитной карты, которая используется в продуктовом магазине или банкомате. Это работает по тому же принципу, что и аудио- или видеопленка, упомянутая в последнем абзаце, в которой голова считывает личную информацию с вашей карты.

Другое применение электромагнитной индукции — это когда электрические сигналы должны передаваться через барьер. Рассмотрим кохлеарный имплант , показанный ниже. Звук улавливается микрофоном на внешней стороне черепа и используется для создания переменного магнитного поля.Ток индуцируется в приемнике, закрепленном в кости под кожей, и передается на электроды во внутреннем ухе. Электромагнитная индукция может использоваться и в других случаях, когда электрические сигналы должны передаваться через различные среды.

Еще одна современная область исследований, в которой успешно применяется (и с большим потенциалом) электромагнитная индукция, — это транскраниальное магнитное моделирование. Множество расстройств, включая депрессию и галлюцинации, можно объяснить нерегулярной локальной электрической активностью в головном мозге.В транскраниальной магнитной стимуляции быстро меняющееся и очень локализованное магнитное поле помещается рядом с определенными участками, идентифицированными в головном мозге. В идентифицированных участках индуцируются слабые электрические токи, которые могут привести к восстановлению электрических функций в тканях мозга.

Апноэ сна («остановка дыхания») поражает как взрослых, так и младенцев (особенно недоношенных детей, и это может быть причиной внезапной детской смерти [SID]). У таких людей дыхание может неоднократно останавливаться во время сна.Прекращение действия более чем на 20 секунд может быть очень опасным. Инсульт, сердечная недостаточность и усталость — вот лишь некоторые из возможных последствий для человека, страдающего апноэ во сне. У младенцев проблема заключается в задержке дыхания на это более длительное время. В одном из типов мониторов, предупреждающих родителей о том, что ребенок не дышит, используется электромагнитная индукция. В проводе, обмотанном вокруг груди младенца, проходит переменный ток. Расширение и сжатие грудной клетки младенца во время дыхания изменяет площадь спирали.В расположенной рядом катушке датчика индуцируется переменный ток из-за изменения магнитного поля исходного провода. Если ребенок перестанет дышать, наведенный ток изменится, и родители могут быть предупреждены.

Установление соединений: сохранение энергии

Закон Ленца — это проявление сохранения энергии. Индуцированная ЭДС создает ток, который противодействует изменению потока, потому что изменение потока означает изменение энергии. Энергия может входить или уходить, но не мгновенно.Закон Ленца — следствие. Когда изменение начинается, закон гласит, что индукция противодействует и, таким образом, замедляет изменение. Фактически, если бы индуцированная ЭДС была в том же направлении, что и изменение потока, была бы положительная обратная связь, которая не давала бы нам бесплатную энергию без видимого источника — закон сохранения энергии был бы нарушен.

Пример 1: Расчет ЭДС: насколько велика индуцированная ЭДС?

Рассчитайте величину наведенной ЭДС, когда магнит, изображенный на Рисунке 1 (а), вдавливается в катушку, учитывая следующую информацию: одноконтурная катушка имеет радиус 6.00 см, а среднее значение [latex] \ boldsymbol {B \; \ textbf {cos} \; \ theta} [/ latex] (это дано, поскольку поле стержневого магнита сложное) увеличивается с 0,0500 Тл до 0,250 Тл. через 0,100 с.

Стратегия

Чтобы найти величину ЭДС , мы используем закон индукции Фарадея, как указано в [latex] \ boldsymbol {\ textbf {emf} = -N \ frac {\ Delta \ phi} {\ Delta t}} [ / latex], но без знака минус, указывающего направление:

[латекс] \ boldsymbol {\ textbf {emf} = N} [/ latex] [латекс] \ boldsymbol {\ frac {\ Delta \ phi} {\ Delta t}} [/ латекс]

Решение

Нам дано, что [латекс] \ boldsymbol {N = 1} [/ latex] и [латекс] \ boldsymbol {\ Delta t = 0.100 \; \ textbf {s}} [/ latex], но мы должны определить изменение потока [latex] \ boldsymbol {\ Delta \ phi} [/ latex], прежде чем мы сможем найти ЭДС. Поскольку площадь петли фиксирована, мы видим, что

[латекс] \ boldsymbol {\ Delta \ phi = \ Delta (BA \; \ textbf {cos} \ theta) = A \ Delta (B \; \ textbf {cos} \; \ theta)} [/ latex]

Теперь [латекс] \ boldsymbol {\ Delta (B \; \ textbf {cos} \; \ theta) = 0.200 \; \ textbf {T}} [/ latex], поскольку было указано, что [латекс] \ boldsymbol { B \; \ textbf {cos} \; \ theta} [/ latex] изменяется с 0.2) (0.200 \; \ textbf {T})} {0.100 \; \ textbf {s}}} [/ latex] [latex] \ boldsymbol {= 22.6 \; \ textbf {mV}} [/ latex]

Обсуждение

Хотя это напряжение легко измерить, его явно недостаточно для большинства практических приложений. Больше петель в катушке, более сильный магнит и более быстрое движение делают индукцию практическим источником напряжения.

.

alexxlab

Добавить комментарий

Ваш адрес email не будет опубликован. Обязательные поля помечены *